You are on page 1of 65

3.1.36 Prove that if G/Z(G) is cyclic then G is abelian.

If G/Z(G) is cyclic with generator xZ(G). Every element in G/Z(G) can be written as xk z
for some k ∈ Z and z ∈ Z(G). Now, let g, h ∈ G, then g = xa z and h = xb w for z, w ∈ Z(G).
We have gh = xa zxb w = xa+b zw = xb+a wz = xb wxa z = hg.

3.2.4 Show that if |G| = pq for some primes p, q (not necessarily distinct), then
either G is abelian or Z(G) = 1.

If |G| = pq and Z(G) 6= 1, we have |G/Z(G)| = p, q or 1. If |G/Z(G)| = 1, then G = Z(G), we


are done. If |G/Z(G)| = p or q, G/Z(G) is cyclic. This implies G is abelian by Exercise 3.1.36.

3.2.8 Prove that if H and K are finite subgroups of G whose orders are relatively
prime then H ∩ K = 1.
|H||K|
Since H ∩ K ≤ H and H ∩ K ≤ K and |HK| = |H∩K| . This implies that |H ∩ K| | |H|,
|H ∩ K| | |K|. Since |H| and |K| are relatively prime, we have |H ∩ K| = 1, hence H ∩ K = 1.

3.2.10 Suppose H and K are subgroups of finite index in the (possible infinite)
group G with |G : H| = m and |G : K| = n. Prove that l.c.m.(m, n) ≤ |G : H ∩ K| ≤ mn.
Deduce that if m and n are relatively prime then |G : H ∩ K| = |G : H| · |G : K|.

Note that H ∩ K is a subgroup of H and K respectively. So there are a, b ∈ Z such that


|G|m |G|m
|H ∩ K| divides a|H| + b|K|. This implies that |H ∩ K| divides a +b . i.e. |H ∩ K|
mn mn
a|G|n + b|G|m an + bm g.c.d.(m, n) 1
divides = |G| = |G| = |G|, which implies that
mn mn mn l.c.m.(m, n)
|G|
|H ∩ K| ≤ . Thus, we have l.c.m.(m, n) ≤ |G : H ∩ K|.
l.c.m.(m, n)
|H||K| |H| |G|
On the other hand, since = |HK| ≤ G by Proposition 13. We have ≤ =
|H ∩ K| |H ∩ K| |K|
|G| |H| |H|
n. Thus, |G : H ∩ K| ≤ · =m· = mn. Therefore, we have
|H| |H ∩ K| |H ∩ K|

l.c.m(m, n) ≤ |G : H ∩ K| ≤ mn

Now, if g.c.d.(m, n) = 1, l.c.m.(m, n) = mn. The above inequality becomes mn ≤ |G :


H ∩ K| ≤ mn. Hence, |G : H ∩ K| = mn = |G : H||G : K|.

3.2.18 Let G be a finite group, let H be a subgroup of G and let N E G. Prove that
if |H| and |G : N | are relatively prime then H ≤ N .

Let f : G → G/N with kernel ker f = N . Consider f |H : H → H/N , then |Imf |H | | |G/N | and
|Imf |H | | |H| (image divides both domain and codomain). But |G/N and |H| are relatively
prime. So Im f |H = 1 which implies that H ≤ N = ker f .

3.2.19 Prove that if N is a normal subgroup of the finite group G and (|N |, |G :
N |) = 1 then N is the unique subgroup of G of order |N |.

1
Let H ≤ G with |H| = |N |. Then (|H|, |G : N |) = (|N |, |G : N |) = 1. By 3.2.18 H ≤ N but
|H| = |N |. So H = N .

3.2.20 If A is an abelian group with A E G and B is any subgroup of G prove that


A ∩ B E AB.

For g ∈ AB, ∈ A ∩ B. So x ∈ A and x ∈ B. Want to show that gxg −1 ∈ A ∩ B. For g ∈ AB,


write g = ab for a ∈ A and b ∈ B. Then (ab)x(ab)−1 = abxb−1 a−1 . Since x ∈ A, A E G and
B ≤ G, we have bxb−1 ∈ A which implies that abxb−1 a−1 ∈ A. Furthermore, since bxb−1 ∈ A
and A abelian, x ∈ B, we have a(bxb−1 )a−1 = aa−1 (bxb−1 ) = bxb−1 ∈ B. Thus, gxg −1 ∈ A∩B
for all g ∈ AB, i.e. A ∩ B E AB.

3.3.3 Prove that if H is a normal subgroup of G of prime index p then for all K ≤ G
either (i) K ≤ H or (ii) G = HK and |K : K ∩ H| = 1.

Since H E G, we have H ≤ HK ≤ G. So p = |G : H| = |G : HK||HK : H|. Thus, either


|HK : H| = 1 or p. In the first case, |HK : H| = 1 implies that HK = H, this gives K ≤ H.
Otherwise, if |HK : H| = p, then |HK : H| = |K : K ∩ H| = p.

3.3.7 Let M and N be normal subgroups of G such that G = M N . Prove that


G/M ∩ N ∼= (G/M ) × (G/N ).

Consider

MN

M N

M ∩N

Define f : G → G/M × G/N by g 7→ (gM, gN ). f is a homomorphism since f (g1 g2 ) =


(g1 g2 M, g1 g2 N ) = (g1 M g2 M, g1 N g2 N ) = (g1 M, g1 N )(g2 M, g2 N ) = f (g1 )f (g2 ). Also ker f =
M ∩ N as follows: for x ∈ ker f , f (x) = (xM, xN ) = (M, N ). Hence x ∈ M and x ∈ N . On
the other hand, for x ∈ M ∩ N , x ∈ M and x ∈ N , clearly x ∈ ker f since f (x) = (M, N ). The
result follows from the first isomorphism theorem.

3.4.1 Prove that G is an abelian simple group then G ∼


= Zp for some prime p (do
not assume G is a finite group).

Suppose that G is infinite. Let x 6= 1 ∈ G be an element of infinite order. Then hxi is a


subgroup of G and actually it must be G since G is simple. So hx2 i is a normal subgroup of
G = hxi (with index 2). This is a contradiction since G is normal.

∼ Zp . If |G| is not prime, say


Now, suppose that G is finite. If the order of G is prime, then G =
|G| = pm for some m. By Cauchy’s Theorem, G has an element of order p, but this implies
2
hxi is a subgroup of G which is a contradiction. Hence, |G| ∼
= Zp by Corollary 10.

In fact, every subgroup of an abelian group is normal. But G is simple, so G has no proper
subgroup. So |G| must be prime, hence G ∼ = Zp .

3.4.5 Prove that subgroups and quotient groups of a solvable groups are solvable.

Let G be a solvable group and H ≤ G a subgroup. Since G is solvable. There is a chain of


subgroups
1 = G 0 E G1 E · · · E Gn = G
such that Gi+1 /Gi is abelian. Then it is clear that H ∩ Gi ≤ H ∩ Gi+1 . And in fact
H ∩ Gi E H ∩ Gi+1 . Since for x ∈ H ∩ Gi and g ∈ H ∩ Gi+1 we have gxg −1 ∈ H. And
gxg −1 ∈ Gi because Gi E Gi+1 . Also it is clear that H ∩ Gi+1 /H ∩ Gi is abelian. Thus H is
solvable.

Similar way shows that G/H is solvable. Since Gi /H ≤ Gi+1 /H and in fact Gi /H E Gi+1 /H.
And (Gi+1 /H)/(Gi /H) is abelian.

3.4.7 If G is a finite group and H E G prove that there is a composition series of


G, one of whose terms is H.

By Theorem 22 (Jordan-Holder Theorem), every finite group has a composition series. So we


have
1 = H0 E H1 E · · · E Hn = H
and
1 = K0 /H E K1 /H E · · · E Km /H = G/H
By Lattice Isomorphism Theorem, this implies that
H = K0 E K1 E · · · E Km = G
Therefore,
1 = H0 E H1 E · · · E H E K1 E · · · E Km = G
3.4.11 Prove that if H is a nontrivial normal subgroup of the solvable group G
then there is a nontrivial subgroup A of H with A E G and A is abelian.

3.4.12 Prove (without using the Feit-Thompson Theorem) that the following are
equivalent:
(i) every group of odd order is solvable.
(ii) the only simple groups of odd order are those of prime order.

(i) ⇒ (ii)

Let G be a simple group of odd order. Suppose that |G| = n = pm for some m 6= 1. If G is
abelian, there exists an element x 6= 1 ∈ G of order p, i.e. hxi E G, a contradiction. If G is
non-abelian, since G is solvable and every finite group has a composition series. So we have
1 = H0 E H1 E · · · E Hn = G
By Exercise 1, the length of the composition series must be at least 2. So Hn−1 6= 1. But this
implies that Hi−1 E G, a contradiction since G is simple. Therefore, the only simple groups of
3
odd order are those of prime order.

(ii) ⇒ (i)

3.5.9. Prove that the (unique) subgroup of order 4 in A4 is normal and is isomor-
phic to V4 .

As shown in the Figure 8 on page 111, the unique subgroup of order 4 in A4 is H =


h(12)(34), (13)(24)i. Let σ = (12)(34) and τ = (13)(24). Since στ = (12)(34)(13)(24) =
(14)(23) and all three elements σ, τ, στ have order 2. So H ∼
= V4 . Now, show that H is nor-
mal. Only need to check on the generators σ, τ and only need to check that gσg −1 ∈ V4 and
gτ g −1 ∈ V4 for g all three cycles containing 3. i.e.
(123)(12)(34)(132) = (14)(23) ∈ V4
(123)(13)(24)(132) = (12)(34) ∈ V4
and
(134)(12)(34)(143) = (14)(23) ∈ V4
(134)(13)(24)(143) = (12)(34) ∈ V4
Thus, H E G.

3.5.10. Find a composition series for A4 . Deduce that A4 is solvable.

Consider the chain of subgroups


1 ≤ h(12)(34)i ≤ V4 ≤ A4
Note that h(12)(34)i E V4 because of index 2. So each subgroup is a normal subgroup of
the group on the right-hand side. And we just proved in previous exercise that V4 E A4 .
Furthermore, the composition factors are
S4 /A4 ∼
= Z2 A4 /V4 ∼ = Z3 V4 /h(12)(34)i ∼
= Z2 h(12)(34)i/1 ∼
= Z2
are all simple. So this is a composition series, hence A4 is solvable.

4.1.1 Let G act on the set A. Prove that if a, b ∈ A and b = g · a for some g ∈ G,
then Gb = gGa g −1 (Ga is the stabilizer of a). Deduce that if G acts transitively on
A then the kernel of the action is ∩g∈G gGa g −1 .

For x ∈ Gb , x · b = b. So g −1 xg · a = g −1 xb = g −1 b = a. Hence g −1 xg ∈ Ga which implies that


x ∈ gGa g −1 . On the other hand, for x ∈ Ga we have gxg −1 · b = gxg −1 ga = gx · a = g · a = b.
Thus, Gb = gGa g −1 .

Let K be the kernel of the action, i.e. K = {g ∈ G | g · a = a for all a ∈ A}. Show that
K = ∩g∈G gGa g −1 .

For x ∈ K, x · a = a. Since g −1 xg · a = g −1 g · a = a for all g ∈ G. We have g −1 xg ∈ Ga for all


g ∈ G. Hence, x ∈ gGa g −1 for all g ∈ G. i.e. x ∈ ∩g∈G gGa g −1 .

On the other hand, let x ∈ ∩g∈G gGa g −1 . Then x ∈ gGa g −1 for all g ∈ G. So for any
b ∈ A we have x · b = ghg −1 · b for some h ∈ Ga (i.e. h · a = a). But G acts transitively on A,
4
i.e. g ·a = b. Thus, x·b = ghg −1 ·b = gh·a = g ·a = b. So x ∈ K. Therefore, K = ∩g∈G gGa g −1 .

4.1.2 Let G be a permutation group on the set A (i.e., G ≤ SA ), let σ ∈ G and let
a ∈ A. Prove that σGa σ −1 = Gσ(a) . Deduce that if G acts transitively on A then
∩σ∈G σGa σ −1 = 1.

Let x ∈ σGa σ −1 . And let x = σgσ −1 for g ∈ Ga . Then x · σ(a) = σgσ −1 · σ(a) = σg · a = σ(a).
Hence x ∈ Gσ(a) . On the other hand, let x ∈ Gσ(a) . So x · σ(a) = σ(a). We have
σ −1 xσ · a = σ −1 (σ(a)) = a. Hence, σ −1 xσ ∈ Ga , i.e., x ∈ σGa σ −1 Therefore σGa σ −1 = Gσ(a) .

Now, if G acts transitively on A, by previous exercise the kernel is ∩g∈G gGg −1 . Let ϕ : G → SA
be the homomorphism associated to the action. Since G ≤ SA , ϕ is injective, the kernel is 1.

4.1.3 Assume that G is an abelian, transitive subgroup of SA . Show that σ(a) 6= a


for all σ ∈ G − {1} and all a ∈ A. Deduce that |G| = |A|. [use the preceding exercise.]

Suppose that σ(a) = a for some a ∈ A and some σ ∈ G. This implies that Ga 6= 1 hence
−1 6= 1, a contradiction to the result in preceding exercise.
T
σ∈G σGa σ

For a fixed a ∈ A, and any b ∈ A, there is σ ∈ G such that σ(a) = b. To show this σ is unique,
suppose there is τ ∈ G such that σ(a) = τ (a) = b. Then τ −1 σ(a) = a, so τ −1 σ ∈ Ga = 1.
Thus, σ = τ . Hence we have an injective map ϕa : A → G sending a to σ. Hence |A| ≤ |G|.
Similarly, if we define ψ : G → A by ψ(σ) = σ · a. Then ψ is injective since σ(a) = τ (a) implies
that τ −1 σ ∈ Ga = 1 which again implies thatσ = τ . Therefore, |G| = |A|.

4.2.8 Prove that if H has finite set index n then there is a normal subgroup K of
G with K ≤ H and |G : K| ≤ n!.

Let G act by left multiplication on the set A of left cosets (i.e. G/H). And let πH be the
associated permutation representation afforded by this action

πH : G → SA

By Theorem 3, K = ker πH = x∈G xHx−1 and K is the normal subgroup of G contained


T
in H. By first isomorphism theorem,
G/K ∼ = ImπH ≤ SA . Since H has index n, |SA | = n!.
G
Hence, we have [G : K] = ≤ |SA | = n!.
K
4.2.9 Prove that if p is a prime and G is a group of order pα for some α ∈ Z+ , then
every subgroup of index p is normal in G. Deduce that every group of order p2
has a normal subgroup of order p.

Since |G| = pα . Since p is the smallest prime dividing |G|. By Corollary 5, any subgroup of
index p is normal. In particular, let α = 2, then |G| = p2 . By Cauchy’s Theorem, G has an
element x of order p. So H = hxi and [G : H] = p. H has index p, hence H is normal.

4.2.11 Let G be a finite group and let π : G → SG be the left regular representation.
Prove that if x is an element of G of order n and |G| = mn, then π(x) is a product
of m n-cycles. Deduce that π(x) is an odd permutation if and only if |x| is even
5
|G|
and |x| is odd.

The permutation representation π : G → SG is defined by π(x) = σx and for each g ∈ G, the


permutation of G is σx (g) = x · g.

Let H = hxi. Since H is cyclic. For each g ∈ G, the stabilizer of g in H is 1 (GH (x) = 1), so
the action is faithful (i.e. the kernel of the action is 1). So each orbit of H in G has order n
and is the cycle containing g in the decomposition of π(x). i.e. for each g,

Hg = (g xg x2 g · · · xn−1 g)

Since |G| = mn, G = Hg1 ∪ · · · ∪ Hgm . Hence the image of x of left regular representation is a
product of m of n-cycles. Now, if |x| is even and |G|
|x| is odd, by Proposition 25 π(x) is an odd
permutation. Conversely, if π(x) is odd and |x| is odd, the number of cycles of even length in
the decomposition is zero, which is even. By Proposition 25 again, π(x) is even, a contradiction.

4.2.12 Let G and π be as in the preceding exercise. Prove that if π(G) contains an
odd permutation then G has a subgroup of index 2. [use Exercise 3 in Section 3.3]

4.2.13 Prove that if |G| = 2k where k is odd then G has a subgroup of index 2. [Use
Cauchy’s Theorem to produce an element of order 2 and then use the preceding
two exercises.]

By Cauchy’s Theorem, there is an element x ∈ G of order 2, so |x| = 2. By Exercise 11, π(x)


is an odd permutation, then by Exercise 12 G has a subgroup of index 2.

4.2.14 Let G be a finite group of composite order n with the property that G has
a subgroup of order k for each positive integer k dividing n. Prove that G is not
simple.

Let p be the smallest prime dividing n. So n = pm for some m ∈ Z. By assumption, G has a


subgroup H of order m. Then H has index p, hence is normal by Corollary 5. Therefore G is
not simple.

4.3.4 Prove that if S ⊆ G and g ∈ G then gNG (S)g −1 = NG (gSg −1 ) and gCG (S)g −1 =
CG (gSg −1 ).

(1) gNG (S)g −1 = NG (gSg −1 )

(⊆) Let gag −1 ∈ NG (gSg −1 ), for a ∈ NG (S) = {x ∈ G | xgSg −1 x−1 = gSg −1 }. We have

(gag −1 )gSg −1 (gag −1 )−1 = gag −1 gSg −1 ga−1 g −1 = gaSa−1 g −1 = gSg −1

since aSa−1 = S. So gag −1 ∈ NG (gSg −1 ).

(⊇) Let x ∈ NG (gSg −1 ). Then xgSg −1 x−1 = gSg −1 . This implies that g −1 xgSg −1 x−1 g = S,
i.e. (g −1 xg)S(g −1 xg)−1 = S. Thus g −1 xg ∈ NG (S), so x ∈ gNG (S)g −1 .

(2) gCG (S)g −1 = CG (gSg −1 )

6
(⊆) Let gag −1 ∈ gCG (S)g −1 , for some a ∈ CG (S) = {x ∈ G | xsx−1 = s for all x ∈ S}. Then
for any s ∈ S we have
(gag −1 )gsg −1 (gag −1 )−1 = gag −1 gsg −1 ga−1 g −1 = gasa−1 g −1 = gsg −1
since asa−1 = s. Thus, gag −1 ∈ NG (gSg −1 ).

(⊇) Let x ∈ CG (gSg −1 ). Then xgsg −1 x−1 = gsg −1 for all s ∈ S. This implies that
g −1 xgsg −1 x−1 g = s, i.e. (g −1 xg)s(g −1 xg)−1 = s for all s ∈ S. Thus g −1 xg ∈ CG (S), so
x ∈ gCG (S)g −1 .

4.3.5 If the center of G is of index n, prove that every conjugacy class has at most
n elements.

By Orbit-Stabilizer Theorem [G : Gg ][Gg : Z(G)] = [G : Z(G)] = n, we see that [G : Gg ] di-


vides n. When the action is conjugation, the number of conjugates of g is precisely |G : CG (g)|.
Thus, |G : CG (g)| divides n, which implies that every conjugacy class has at most n elements.

4.3.6 Assume G is a non-abelian group of order 15. Prove that Z(G) = 1. Use the
fact that hgi ≤ CG (g) for all g ∈ G to show that there is at most one possible class
equation for G. [Use Exercise 36, Section 3.1]

Center Z(G) is a subgroup of G. Since G is non-abelian, |Z(G)| can only be 1, 3, 5. Suppose


|Z(G)| = 5. By Orbit- Stabilizer Theorem [G : Gg ][Gg : Z(G)] = [G : Z(G)] = 15/5 = 3. So
the number of conjugates of any non-center element g ∈ G, i.e. |G : CG (g)|, must divides 3.
And it must be 3, otherwise g would be in the center. However, by Class Equation,
r
X
|G| = |Z(G)| + |G : CG (gi )|
i=1
Pr
we have i=1 |G : CG (gi ) = 15 − 5 = 10 which is not divisible by 3, a contradiction.

A quicker way: suppose |Z(G)| is prime, by Exercise 36 in Section 3.1, G/Z(G) is cyclic, so G
is abelian, a contradiction. Hence |Z(G)| = 1.

Now, since hgi ≤ CG (g) for all g ∈ G (Exercise 6 in Section 2.2). Let g ∈ G be any non-identity
element in G. Since Z(G) = 1 hgi ≤ CG (g) G. |CG (g)| can only be 3 or 5. Since |G| = 15,
by Lagrange Theorem, we must have hgi = P CG (g). So the number of conjugates in every
conjugacy class is 3 or 5. By Class Equation, ri=1 |G : CG (gi )| = 15 − 1 = 14. Thus, we have
the relation 3a + 5b = 14 where a denotes the number of conjugacy classes of order 3 and b
denotes the number of conjugacy classes of order 5. The only one solution is a = 3 and b = 1.
Therefore, there is at most one possible class equation for G. i.e. |G| = 15 = 1 + 3 + 3 + 3 + 5.

4.3.13 Find all finite groups which have exactly two conjugacy classes.

Every element in an abelian group is a conjugacy of its own. So abelian groups having exactly
two conjugacy classes must have two element hence is isomorphic to Z2 . Now, let G be non-
abelian group. Note that the identity in G forms a conjugacy class of its own. So |Z(G)| = 1.
Since we assume that G has exactly two conjugacy classes, there is an element g 6∈ Z(G) such
that
|G| = 1 + |G : CG (g)|
7
by class equation. And |G : CG (g)| is the number of elements in the other conjugacy class. So
we have
|G| − 1 = |G : CG (g)|
this implies that the number of conjugates in the other conjugacy class divides |G| − 1. But
this happens only when |G| = 2. Hence G is isomorphic to Z2 .

4.3.23 Recall that a proper subgroup M of G is called maximal if whenever


M ≤ H ≤ G, either H = M or H = G. Prove that if M is a maximal subgroup
of G then either NG (M ) = M or NG (M ) = G. Deduce that if M is a maximal sub-
group of G that is not normal in G then the number of non-identity elements of G
that are contained in conjugates of M is at most (|M | − 1)|G : M |.

Since M ≤ NG (M ) ≤ G by Exercise 6 in Section 2.2 and the fact that NG (M ) is a subgroup


of G. But M is a maximal subgroup of G, so it is clear that either NG (M ) = M or NG (M ) = G.

If M is not normal, i.e. NG (M ) G, then M = NG (M ). So we have



G G
NG (M ) = M

By Proposition 6, the number of conjugates of M , gM g −1 , is [G : NG (M )]. And each conju-


gate of M has cardinality |gM g −1 | = |M |. The largest number of non-identity elements in the
conjugates of M occurs when these conjugates intersect trivially. Hence, the number of non-
identity elements of G that are contained in the conjugates of M is at most (|M | − 1)|G : M |.

4.3.24 Assume H is a proper subgroup of the finite group G. Prove that G 6=


∪g∈G gHg −1 , i.e. G is not the union of the conjugates of any proper subgroup. [Put
H in some maximal subgroup and use the preceding exercise.]

Let M be a maximal subgroup of G containing H. If M is normal in G, we have ∪g∈G gHg −1 ⊆


∪g∈G gM g −1 = M 6= G. If M is not normal in G, we still have ∪g∈G gHg −1 ⊆ ∪g∈G gM g −1 . By
previous exercise, we know that ∪g∈G gM g −1 contains at most (|M | − 1)|G : M | non-identity
elements of G. Hence,
[ |G|
| gHg −1 | ≤ (|M | − 1)|G : M | = |G| − + 1 < |G| < ∞
|M |
g∈G

|G|
since > 1.
|M |
4.3.27 Let g1 , g2 , · · · , gr be representatives of conjugacy classes of the finite group
G and assume these elements pairwise commute. Prove that G is abelian.

If g1 , g2 , · · · , gr are pairwise commute, then gi gj = gj gi which implies that gj gi gj−1 = gi . Thus,


for each i, we have gj ∈ CG (gi ) for all j = 1, · · · , r, so |C(gi )| ≥ r. By Class Equation, |G| is
the sum of all distinct conjugacy classes. Each conjugacy class has |G : CG (gi )| elements, and
there are r of them. Hence
r r r
X X |G| X |G|
|G| = |G : CG (gi )| = ≤ = |G|
|CG (gi )| r
i=1 i=1 r=1
8
This implies that |CG (gi )| = r. i.e. CG (gi ) = {g1 , g2 , · · · , gr }. We claim that G = CG (gi ).
G ⊇ CG (gi ) is clear. And every element x ∈ G is in the centralizer of itself, hence G ⊆ CG (gi ).
Therefore, G = CG (gi ), hence G is abelian.

4.3.29 Let p be a prime and let G be a group of order pα . Prove that G has a
subgroup of order pβ , for every β with 0 ≤ β ≤ α. [Use Theorem 8 and induction
on α.]

Suppose |G| = p, then G has a subgroup order 1 and p, so this is clear.

Suppose any group of order pa has a subgroup of order pb for every b with 0 ≤ b ≤ a. Let G
be a group of order |G| = pa+1 , want to prove that G has a subgroup of order pb for every
0 ≤ b ≤ a + 1. By Theorem 8, Z(G) 6= 1. Then H = Z(G) is a normal subgroup of G. For
any subgroup A of G containing H, we have the following correspondence:

G/H /G

 
A/H /A

 
1 /H

Since |G/H| = pc for some c with 0 ≤ c ≤ b. By induction hypothesis, G/H has subgroup of
order pb for every b with 0 ≤ b ≤ c. Hence by Lattice Isomorphism Theorem, G has a subgroup
of order pb for every 0 ≤ b ≤ n + 1.

4.3.30 If G is a group of odd order, prove for any nonidentity element x ∈ G that
x and x−1 are not conjugate in G.

First, note that x 6= x−1 . since if x = x−1 , then x2 = 1 and |x| = 2, a contradiction since |G|
is even and cannot have an element of even order.

Suppose x and x−1 are conjugate, want to show that CG (x) has even order, a contradiction.
By definition, CG (x) = {g ∈ G | gxg −1 = x}.

4.4.13 Let G be a group of order 203. Prove that if H is a normal subgroup of


order 7 in G then H ≤ Z(G). Deduce that G is abelian in this case.

|G| = 203 = 7 · 29. Since H is normal, by Corollary 13 |G/CG (H) is isomorphic to a subgroup
of Aut(H). And |Aut(H)| = 6, so |G/CG (H)| must divide 6. Since |G| = 7 · 29, the only
possibility is |G/CG (H)| = 1. Hence G = CG (H). i.e. H ≤ Z(G). So |H| divides |Z(G)|.
Since |P | = 7, we have |Z(G)| = 7 or 7 · 29. This implies that |G/Z(G)| = 1 or 29, hence
G/Z(G) is cyclic. Therefore, G is abelian.

4.5.13 Prove that a group of order 56 has a normal Sylow p-subgroup for some
prime p dividing its order.

9
|G| = 56 = 23 · 7. Since
n2 ≡ 1 (mod 2), n2 | 7 so n2 = 1, 7
n7 ≡ 1 (mod 7), n2 | 8 so n7 = 1, 8
If n7 = 1, the Sylow 7-subgroup is normal in G. Then we are done. If n7 = 8, G has 8 · 6 = 48
non-identity elements of order 8 in these Sylow 7-subgroups. The rest 56 − 48 = 8 are precisely
elements in Sylow 2-subgroup. This implies that G has a unique Sylow 2-subgroup.

4.5.14 Prove that a group of order 312 has a normal Sylow p-subgroup for some
prime p dividing its order.

|G| = 56 = 23 · 3 · 13. Since


n3 ≡ 1 (mod 3), n3 | 8 · 13 so n3 = 1, 4, 13, 52
n13 ≡ 1 (mod 13), n13 | 8 · 13 so n13 = 1
So Syl13 is normal in G.

4.5.15 Prove that a group of order 351 has a normal Sylow p-subgroup for some
prime p dividing its order.

|G| = 351 = 33 · 13. Since


n3 ≡ 1 (mod 3), n3 | 13 so n3 = 1, 13
3
n13 ≡ 1 (mod 13), n13 | 3 so n13 = 1, 27
If n13 = 1, the Sylow 13-subgroup is normal in G. Then we are done. If n13 = 27, G
has 27 · 12 = 324 non-identity elements of order 13 in these Sylow 13-subgroups. The rest
351 − 324 = 27 are precisely elements in Sylow 3-subgroup. This implies that G has a unique
Sylow 3-subgroup.

4.5.17 Prove that if |G| = 105 then G has a normal Sylow 5-subgroup and a normal
Sylow 7-subgroup.

(The solution basically follow the proof of the example on page 143 where |G| = 30).
|G| = 105 = 3 · 5 · 7. Since
n5 ≡ 1 (mod 5), n5 | 3 · 7 so n5 = 1, 21
n7 ≡ 1 (mod 7), n7 | 3 · 5 so n7 = 1, 15
Let P = Syl5 and Q = Syl7 . If n5 = 1 and n7 = 1, then we are done. If either n5 = 1 or n7 = 1,
i.e., either P or Q is normal. Let H = P Q. Then H ≤ G by Corollary 15, and both P and Q
are characteristic subgroups of H by statement (2) on page 135 that every subgroup of a cyclic
group is characteristic (H is cyclic by Example on page 143), or it is easy to see that n5 = 1 and
n7 = 1, hence P EH and QEH. So P and Q are characteristic in H. Since H EG by Corollary
5. By exercise 4.4.8 or the statement (3) on page 135, both P and Q are normal subgroup of G.

Now, assume that neither Sylow subgroup is normal, i.e., n5 = 21 and n7 = 15. Then G has
21 · 4 = 84 elements of order 5 and 15 · 6 = 90 elements of order 7. So total elements would be
84 + 90 = 174 > 105, a contradiction. Therefore, one of P or Q, hence both, must be normal
in G.

10
4.5.24 Prove that if G is a group of order 231 then Z(G) contains a Sylow 11-
subgroup of G and a Sylow 7-subgroup is normal in G.

|G| = 231 = 3 · 7 · 11. Since


n3 ≡ 1 (mod 3), n3 | 7 · 11 so n3 = 1
n7 ≡ 1 (mod 7), n7 | 3 · 11 so n7 = 1
n11 ≡ 1 (mod 11), n11 | 3 · 7 so n11 = 1
It is clear that Syl7 E G.

Let P = Syl11 . Since G/CG (P ) ∼


= a subgroup of Aut(P ) by Corollary 13. And |Aut(P )| =
11 − 1 = 10, so |G/CG (P )| must divides 10. But |G| = 3 · 7 · 11, the only possibility is that
|G/CG (P )| = 1. Hence, G = CG (P ). i.e. P ≤ Z(G).

4.5.25 Prove that if G is a group of order 385 then Z(G) contains a Sylow 7-
subgroup of G and a Sylow 11-subgroup is normal in G.

|G| = 385 = 5 · 7 · 11. Since


n5 ≡ 1 (mod 5), n5 | 7 · 11 so n5 = 1, 11
n7 ≡ 1 (mod 7), n7 | 5 · 11 so n7 = 1
n11 ≡ 1 (mod 11), n11 | 5 · 7 so n11 = 1
It is clear that Syl11 E G.

Let P = Syl7 . Since G/CG (P ) ∼= a subgroup of Aut(P ) by Corollary 13. And |Aut(P )| = 6.
So |G/CG (P )| must divides 6. But |G| = 5 · 7 · 11, the only possibility is that |G/CG (P )| = 1.
Hence, G = CG (P ). i.e. P ≤ Z(G).

4.5.26 Let G be a group of order 105. Prove that if a Sylow 3-subgroup of G is


normal then G is abelian.

|G| = 385 = 3 · 5 · 7. Since


n3 ≡ 1 (mod 3), n3 | 5 · 7 so n3 = 1, 7
n5 ≡ 1 (mod 5), n5 | 3 · 7 so n5 = 1, 21
n7 ≡ 1 (mod 7), n7 | 3 · 5 so n7 = 1, 15
If P = Syl3 is normal. By Corollary 13, G/CG (P ) is isomorphic to a subgroup of Aut(P ).
Since |Aut(P )| = 2, so |G/CG (P )| must divides 2. But |G| = 3 · 5 · 7, the only possibility is
that |G/CG (P )| = 1. Hence, G = CG (P ). i.e. P ≤ Z(G). Similarly, Q = Syl5 is normal by
Exercise 17. Thus we can also prove that Q ≤ Z(G). By Lagrange, |Z(G)| = 3 · 5 or 3 · 5 · 7.
So |G/Z(G)| = 1, 7 hence G/Z(G) is cyclic. Therefore, G is abelian.

4.5.27 Let G be a group of order 315 which has normal Sylow 3-subgroup. Prove
that Z(G) contains a Sylow 3-subgroup of G and deduce that G is abelian.

|G| = 315 = 32 · 5 · 7. Since


n3 ≡ 1 (mod 3), n3 | 5 · 7 so n3 = 1, 7
2
n5 ≡ 1 (mod 5), n5 | 3 · 7 so n5 = 1, 21
11
n7 ≡ 1 (mod 7), n7 | 32 · 5 so n7 = 1, 15
If Sylow 3-subgroup, P = Syl3 , is normal, G/CG (P ) is isomorphic to a subgroup of Aut(P ) by
Corollary 13. Since |Aut(P )| = 6 or 48. And P is a square of a prime, P is an abelian group,
hence P ≤ CG (P ) by Exercise 6(b) in Section 2.2. It follows that |CG (P )| is divisible by 9,
which implies that |G/CG (P )| = 1, 5, 7, 35. Together these imply |G/CG (P )| = 1. Hence,
G = CG (P ). i.e. P ≤ Z(G). So |G/Z(G)| =1, 5, 7 or 35. If |G/Z(G)| = 1, then G is abelian.
If |G/Z(G)| = 5 or 7, G/Z(G) is cyclic hence G is abelian. If |G/Z(G)| = 35, it is easy to see
that n5 = 1 and n7 = 1 then G/Z(G) is cyclic by Example on page 143. Therefore, G is abelian.

4.5.32 Let P be a Sylow p-subgroup of H and let H be a subgroup of K. Let P E H


and H EK, prove that P is normal in K. Deduce that if P ∈ Sylp (G) and H = NG (P ),
then NG (H) = H.

If P is a Sylow p subgroup of H and assume that P E H, then P is characteristic in H by


Corollary 20. Since H is normal in K, by the statement (3) P is normal in K. Hence, if
P ∈ Sylp (G) and H = NG (P ) (i.e., P E H), then by above argument P E K. This implies that
K = NK (P ) ≤ NG (P ) = H
Thus, H = K. Since H E K (i.e. K = NG (H)), we have H = NG (P ) E K which implies that
NG (H) = NG (NG (P )) = K = NG (P ) = H, as desired. (the solutions was not correct, will
come back later!)

4.5.33 Let P be a normal Sylow p-subgroup of G and let H be any subgroup of G.


Prove that P ∩ H is the unique Sylow p-subgroup of H.

It suffices to show that P ∩ H is Sylow p subgroup of H. Then by Exercise 24 in Section 3.1,


P ∩ H E H.

Let |G| = pα m with p - m. Then |H| = pβ m where 0 ≤ β ≤ α and |P | = pα . Since P E G,


HP ≤ G. By Proposition 13 in Section 3.2, we have
|H| |HP | |G| pα m
= divides = α =m
|H ∩ P | |P | |P | p
Hence |P ∩ H| = pβ . Thus, P ∩ H is the unique Sylow p-subgroup of H.

Alternatively, since P ∩ H E H and P ∩ H is a p-subgroup of H. Suppose that P ∩ H is not a


Sylow p-subgroup of H. There is an element x ∈ H\P of order p. But this is not possible since
P is the unique Sylow p-subgroup of G, that contains all elements of order p, a contradiction!

4.5.34 Let P ∈ Sylp (G) and assume N E G. Use the conjugacy part of Sylow’s Theo-
rem to prove that P ∩ N is a Sylow p-subgroup of N . Deduce that P N/N is a Sylow
p-subgroup of G/N

Suppose Q is a Sylow p-subgroup of N . Q is also a p-subgroup of G, by Theorem 18(2), there


exists g ∈ G such that Q ⊆ gP g −1 . Since Q ⊆ N , we have Q ⊆ gP g −1 ∩ N . But N is normal.
so Q ⊆ gP g −1 ∩ gN g −1 = g(P ∩ N )g −1 . This implies that g −1 Qg ⊆ P ∩ N . Now, g −1 Qg
is a Sylow p-subgroup of N since |g −1 Qg| = |Q|. The group P ∩ N is a p-subgroup of N , so
g −1 Qg = P ∩ N . This shows that P ∩ N is a Sylow p-subgroup of N .

12
Let |P | = pa and |Q| = pb , so a Sylow p-subgroup of G/N has order pa−b . By 2nd Isomorphism
Theorem,
PN P
=
N P ∩N
So |P N/N | = |P |/|P ∩ N | = pa /pb = pa−b . Thus, P N/N is a Sylow p-subgroup of G/N .

G/N G

P N/N PN

1 N
10.1.5 For any left ideal I of R define
X
IM = { ai mi | ai ∈ I, mi ∈ M }
finite
to be the collection of all finite sums of elements of the form am where a ∈ I and
m ∈ M . Prove that IM is a submodule of M .
First, prove that IM is a subgroup. PNote that thePidentity exists since 0 ∈P
M and take m = 0,
then 0m ∈ IM . P For x, y ∈ IM , x = mi , y = bi mi , we have x + y = (ai + bi )mi ∈ IM .
aiP
Finally, let y = −ai mi , then x + y = (ai − ai )mi = 0, so y is the inverse of x. Thus, IM
is a subgroup.
P P
IM is closed under the action of ring elements: let r ∈ R, rx = r ai mi = rai mi ∈ IM
since rai ∈ I. Hence, IM is a submodule.

10.1.7 Let N1 ⊆ N2 ⊆ · · · be an ascending chain of submodules of M . Prove that


∪∞
i=1 Ni is a submodule of M .

We prove by Proposition 1. First, ∪∞


i=1 Ni 6= φ since Ni 6= φ for each i. Furthermore, for r ∈ R
and x, y ∈ ∪∞ N
i=1 i , we have x ∈ N m and y ∈ Nk for some m, k. Without loss of generality,
assume m ≤ k. Then x ∈ Nk and x + ry ∈ Nk hence x ∈ ∪∞ i=1 Ni .

10.1.9 If N is a submodule of M , the annihilator of N in R is defined to be


{r ∈ R | rn = 0 for all n ∈ N }. Prove that the annihilator of N in R is a 2-sided
ideal of R.

(1) Prove that Ann(N ) is a subring.


Note that 0 ∈ N , so Ann(N ) is nonempty. For x, y ∈ Ann(N ), xn = 0 and yn = 0 for
all n ∈ N . So (x − y)n = xn − yn = 0 for all n ∈ N . Hence, x − y ∈ Ann(N ). Now,
(xy)n = x(yn) = x · 0 = 0. Hence Ann(N ) is a subring.

(2) Prove that Ann(N ) is closed under right and left multiplication by element of r ∈ R.
Let x ∈ Ann(N ), xn = 0 for all n ∈ N . (rx)n = r(xn) = r · 0 = 0, hence rx ∈ Ann(N ).
Similarly, (xr)n = x(rn) = x · 0 = 0.

Therefore, Ann(N ) is a 2-sided ideal of R.

13
10.1.19 Let F = R, let V = R2 and let T be the linear transformation from V to V
which is projection onto the y-axis. Show that V , 0, the x-axis and the y-axis are
the only F [x]-submodules for this T .

Since the F [x]-submodules of V are precisely the T -invariant subspace of V . We see that
T (0) = 0 ⊆ V , T (V ) ⊆ V , T (x-axis) = 0 ⊆ x-axis and T (y-axis) = y-axis ⊆ y-axis. Hence,
these are F [x]-modules.

In general, for L be any 1-dimensional subspace on R2 , neither x-axis nor y-axis. It is easy to
see that T (L) 6⊆ L. Hence the only F [x]-submodules for T are 0, V , x-axis and y-axis.

10.2.4 Let A be any Z-module, let a be any element of A and let n be a positive
integer. Prove that the map ϕa : Z/nZ → A given by ϕa (k̄) = ka is a well-define
Z-module homomorphism if and only if na = 0. Prove that HomZ (Z/nZ, A) ∼ = An ,
where An = {a ∈ A | na = 0} (so An is the annihilator in A of the ideal (n) of Z).

Since ka = ma if and only if (k − m)a = 0 if and only if a ∈ nZ if and only if na = 0.


This proves that ϕa is well-defined if and only if na = 0. For x̄, ȳ ∈ Z/nZ, r ∈ Z, we have
ϕa (x̄ + ȳ) = (x + y)a = xa + ya = ϕa (x̄) + ϕa (ȳ). And ϕa (rx̄) = rxa = rϕa (x̄). Hence, ϕa is
Z-module homomorphism.

Define f : Hom(Z/nZ, A) → An by f (ϕa ) = ϕa (k̄) = ka for k ∈ Z/nZ. First, f is well-defined


since ϕa is. For ϕa , ϕb ∈ Hom (Z/nZ, A), we have f (ϕa + ϕb )(k̄) = ϕa (k̄) + ϕb (k̄) = ka + kb =
ϕa (k̄) + ϕb (k̄) = f (ϕa ) + f (ϕb ). Furthermore, let r ∈ R, then we have f (rϕa ) = rϕa (k̄) =
rka = r(ka) = rϕa (k̄) = rf (ϕa ). So f is a group homomorphism. To show that f is surjective,
for a ∈ An with na = 0, let k̄ = 1, then we have ϕa (1̄) = 1 · a = a. Suppose ϕa (k̄) = ϕb (k̄),
then ka = kb which implies that a = b (by cancellation property since A is an abelian group).
i.e. ϕa = ϕb . Therefore, f is an isomorphism.

10.2.6 Prove that HomZ (Z/nZ, Z/mZ) ∼


= Z/(n, m)Z.

The homomorphism between them will be determined by where the generator in Z/nZ goes.
Let ϕ : Z/nZ → Z/mZ be a Z-module homomorphism, then ϕ(0) = 0. Hence,

0 = ϕ(0) = ϕ(1 + 1 + · · · + 1) = ϕ(1) + · · · + ϕ(1) = nϕ(1)

To have a homomorphism, we must have nϕ(1) ∈ mZ and this is true if and only ϕ(1) =
m m
, or its multiples. These elements form a group generated by and is cyclic
gcd(m, n) gcd(m, n)
of order gcd(m, n). Therefore, HomZ (Z/nZ, Z/mZ) ∼ = Z/(m, n)Z.

10.2.7 Let z be a fixed element of the center of R. Prove that the map m 7→ zm is
an R-module homomorphism from M to itself. Show that for a commutative ring
R the map from R to EndR (M ) given by r 7→ rI is a ring homomorphism (where I
is the identity endomorphism).

Let ϕ : R × M → M be defined by ϕ(m) = zm. For x, y ∈ M and r ∈ R, we have


ϕ(rx + y) = z(rx + y) = z(rx + y) = zrx + zy = rzx + zy = rϕ(x) + ϕ(y). Hence, ϕ is
an R-module homomorphism.

14
Recall that EndR (M ) =HomR (M, M ). Let ψ : R →EndR (M ) be defined by ψ(r) = rI. For
r, s ∈ R, we see that ψ(r + s) = (r + s)I = rI + sI = ψ(r) + ψ(s). Also, ψ(rs) = rsI = rI · sI =
ψ(r)ψ(s). Therefore, ψ is a R homomorphism.

10.2.11 Let A1 , A2 , · · · , An be R-modules and let Bi be submodule of Ai for each


i = 1, 2, · · · , n. Prove that (A1 × · · · × An )/B1 × · · · × Bn ) ∼
= (A1 /B1 ) × · · · × (An /Bn ).

Define a map ϕ : A1 × · · · × An −→ (A1 /B1 ) × · · · × (An /Bn ) by (a1 , · · · , an ) 7→ (a1 +


B1 , · · · , an + Bn ) for ai ∈ Ai , i = 1, 2, · · · n. Since on each component Ai −→ Ai /Bi is a
natural projection hence is an R-module homomorphism by Proposition 3. Therefore, ϕ is an
R-module homomorphism.

Prove that ϕ is a bijection.

First, note that ϕ is surjective since each component ϕi : Ai → Ai /Bi is a natural projection
hence surjective (well, I thought it is clear that the product of a surjective map is surjective).
It suffices to prove that ker ϕ = B1 × · · · × Bn .

(1) ker ϕ ⊆ B1 × · · · × Bn
If a = (a1 , · · · , an ) ∈ ker ϕ, then ϕ(a) = (a1 + B1 , · · · , an + Bn ) = (0̄, · · · , 0̄) ∈ (A1 /B1 ) ×
· · · × (An /Bn ). So we have ai + Bi = 0̄ hence ai ∈ Bi for each i. Thus, a = (a1 , · · · , an ) ∈
B1 × · · · × Bn .

(2) ker ϕ ⊇ B1 × · · · × Bn
For b = (b1 , · · · , bn ) ∈ B1 × · · · × Bn , we have ϕ(b) = ϕ(b1 , · · · , bn ) = (b1 + B1 , · · · , bn + Bn ) =
(B1 , · · · , Bn ) = (0̄, · · · , 0̄) in (A1 /B1 ) × · · · × (An /Bn ). Hence, b ∈ ker ϕ.

Therefore, ker ϕ = B1 × · · · × Bn . By first isomorphism theorem for modules (Theorem 4 in


Section 2), (A1 × · · · × An )/B1 × · · · × Bn ) ∼
= (A1 /B1 ) × · · · × (An /Bn ).

10.2.13 Let I be a nilpotent ideal in a commutative ring R, let M and N be R-


modules and let ϕ : M → N be an R-module homomorphism. Show that if the
induced map ϕ̄ : M/IM → N/IN is surjective, then ϕ is surjective.

ϕ
M /N

p q
 ϕ̄ 
M/IM / N/IN

Given n ∈ N , want to show that there is m ∈ M such that ϕ(m) = n by showing that
q(ϕ(m) − n) = 0̄ ∈ N/IN . Indeed, by commutativity of the diagram we have

q(ϕ(m) − n) = q · ϕ(m) − q(n)


= ϕ̄ · p(m) − q(n)
= ϕ̄(m + IM ) − q(n)
= n + IN − (n + IN )
= 0̄
15
Thus, ϕ(m)−n ∈ IN . This is true for all n ∈ N , hence ϕ(M )−N = IN . i.e. N = ϕ(M )+IN .
This implies that
N = ϕ(M ) + I(ϕ(M ) + IN )
= ϕ(M ) + Iϕ(M ) + I 2 N
= ϕ(M ) + I 2 N
Inductively, we obtain N = ϕ(M ) + I r N . Since I is nilpotent, I k = 0 for some k ≥ 1. There-
fore, N = ϕ(M ) and ϕ is surjective.

10.3.2 Assume R is commutative. Prove that Rn ∼ = Rm if and only if n = m, i.e.


two free R-modules of finite rank are isomorphic if and only if they have the same
rank. [Apply Exercise 12 of Section 2 with I a maximal ideal of R. You may
assume that F is a field, then F n ∼
= F m if and only if n = m.]

By Exercise 12 in Section 10.2, we have Rn /IRn ∼= I/IR × · · · × I/IR. If I ⊂ R is a maximal


ideal, then R/I is a field. Also, IR = I. Hence we have
Rn /IRn ∼
= R/I × · · · × R/I
Let F = R/I we obtain
Rn /IRn ∼
= F × ··· × F ∼
= Fn
Similarly, we have
Rm /IRm ∼
= R/I × · · · × R/I ∼
= Fm
Therefore,
m = n ⇔ F m = F n ⇔ Rm /IRm = Rn /IRn ⇔ Rm = Rn
10.3.7 Let N be a submodule of M . Prove that if both M/N and N are finitely
generated then so is M .

Let f : M → M/N be a projection homomorphism defined by m 7→ m + N . Let {y1 , · · · , yn }


be a basis for N andP{x1 , · · · , xm } be a basis for M/N . For
P m ∈ M , its image P m̄ in M/N can
be written
P as m̄ = i a i x̄ i . This implies that
P m + N =
P i ai (x i + N ) = Pi ai xi + NP
. Thus,
m − i ai xi ∈ N , which implies that m − i ai xi = j bj yj . Thus, m = i ai xi + j bj yj .
Note that M ∼ = N ⊕ M/N , therefore M is finitely generated.

10.3.9 (a) Show that M is irreducible if and only if M 6= 0 and M is cyclic module
with any nonzero element as generator.

(⇒)
Suppose M is not cyclic, let M = ha1 , · · · , an i for n > 1. i.e. M is finitely generated by
a1 , · · · , an with ai 6= 0 for all i. Then M has a submodule N 6= 0, and N ( M . This is because
we can let A = {a1 , · · · , ak } for k < n, then N = RA = {r1 a1 , · · · , rk ak | ri ∈ R, ai ∈ A} is a
submodule. This proved that M is cyclic.

Now, wuppose M is irreducible, pick a nonzero element m ∈ M , then 0 6= Rm ⊂ M . Clearly


Rm is a submodule. Hence Rm = M since M is irreducible. Therefore, M is a cyclic module.

(⇐)
Suppose M is a cyclic module with any nonzero element as generator. i.e. M = Ra for any
a ∈ M . Want to prove that M is irreducible. Assume that M has a proper submodule N , then
16
0 6= N ⊂ M . Pick a nonzero element n in N , so we have 0 6= Rn ⊂ N ⊂ M . But Rn = M , so
N = M hence M is irreducible.

Now, determine all the irreducible Z-modules.


Let M be an irreducible Z-module. Note that Z-module can be seen as an abelian group, so
by the structure of an abelian group, M has torsion part and free part. Since M is irreducible
hence does not have any proper submodule. But Tor(M ) is a submodule of M . So Tor(M ) is
either 0 or Tor(M ) = M .

Suppose Tor(M )=0, take m ∈ M and m 6= 0. We have am 6= bm for a, b ∈ Z, a 6= b. So we


have 0 6= 2Zm ( Zm ⊂ M . Contradiction to the fact that M is irreducible.

On the other hand, suppose Tor(M ) = M , take m ∈ M and m 6= 0. By what we just proved in
(⇒), M = Zm. Now, since Z is a PID, so Ann(m) = (k) for some k ∈ Z. Then M ∼ = Z/kZ. We
claim that k is a prime. If not, k = qr for some q, r ∈ Z, then 0 6= Z/qZ ( Z/kZ ⊂ M . Con-
tradiction to the fact that M is irreducible. So k is a prime. Hence we proved that M ∼
= Z/pZ
some some prime p.

Finally, if p is a prime, by what we proved in (⇐), Z/pZ is irreducible.

10.3.10 Assume R is commutative. Show that an R-module M is irreducible if and


only if M is isomorphic (as a R-module) to R/I where I is a maximal ideal of R.

(⇒)
Suppose M is irreducible. Define ϕ : R → M by r 7→ rm for m 6= 0, by previous exercise, M
is cylic, i.e. M = Rm for some m 6= 0. So ϕ is actually surjective. Prove that ker ϕ =Ann(m).
For r ∈ ker ϕ, ϕ(r) = rm = 0 implies that r ∈Ann(m). The converse is also clear. Thus we
have R/I ∼ = M where I = ker ϕ =Ann(M ).

Now, prove that I is a maximal ideal. First, I is an ideal since for any s ∈ R, s(rm) = s · 0 = 0.
So SI ⊆ I. To show that I is maximal, suppose I is not a maximal ideal, there is an ideal J
such that I ( J ( R.

R/I ∼
=M R

J/I J

1 I

Then M ∼= R/I has a submodule J/I which contradicts to the irreducibility of M . Hence, I
must be a maximal ideal.

(⇐)
Suppose M ∼ = R/I for a maximal ideal I of R. If M is not irreducible, there is a submodule
N such that N ( M . By Lattice Theorem again, there is an ideal J with I ( J ( R. This
implies that J is not maximal.

17
10.3.11 If M1 and M2 irreducible R-modules, prove that any nonzero R-module
homomorphism from M1 to M2 is an isomorphism.

Let ϕ : M1 −→ M2 be a nonzero R-module homomorphism. Want to prove that ϕ is surective


and injective hence is a bijection. First, note that ker ϕ is a submodule of M1 and im ϕ is a
submodule of M2 by 10.2.1 in last homework assignment. But M1 and M2 are irreducible, so
ker ϕ = 0 (note that ker ϕ can not be M , or it would imply that ϕ is a zero map which contra-
dicts to the assumption), so ϕ is injective. On the other hand, we obtained that im ϕ = M2
(note that im ϕ 6= 0, if it were, ϕ is a zero map which again contradicts to the assumption).
This implies that ϕ is surjective. Therefore, ϕ is a bijection, hence is an isomorphism.

Now, if M is irreducible, prove that EndR (M ) is a division ring.

Let ϕ : M −→ M be a nonzero R-module homomorphism. So ϕ ∈ EndR (M ). But we have just


proved that ϕ is a bijection in this case, which implies that ϕ has an inverse. In other words,
every nonzero element in EndR (M ) has a multiplicative inverse. Note that the “multiplication”
in EndR (M ) is the composition. i.e. there exists ψ ∈ EndR (M ) such that ϕ ◦ ψ = ψ ◦ ϕ = 1M .

Therefore, EndR (M ) is a division ring.

10.4.4 Show that Q ⊗Z Q and Q ⊗Q Q are isomorphic left Q-modules. [Show they
are both 1-dimensional vector spaces over Q.]

By definition on page 367, since Q is a left Z-module and a (Q, Z) bimodule, hence Q ⊗Z Q is
a left Q-module. Also, since Q is a left Q-module and a (Q, Q) bimodule, thus Q ⊗Q Q.

Now, show that they are isomorphic to 1-dimensional vector spaces over Q.

For a ⊗ b ∈ Q ⊗Q Q for a, b ∈ Q, we have a ⊗ b = a · 1 ⊗ b = a(1 ⊗ b) = a(1 ⊗ b · 1) = a(1 · b ⊗ 1) =


a(b · 1 ⊗ 1) = ab(1 ⊗ 1). Thus, Q ⊗Q Q is SpanQ {1 ⊗ 1} ∼ = Q. Hence, Q ⊗Q Q is a 1-dimensional
vector space over Q.

On the other hand, since Q ⊗Z Q is a left Q-module defined by


X X
s( a ⊗ b) = sa ⊗ b for s ∈ Q and a, b ∈ Q
So
p 1 ap 1 p
a⊗b=a⊗ = ap ⊗ = q ⊗ = a ⊗ 1 = ab ⊗ 1
q q q q q
Thus, Q ⊗Z Q is a group generated by α ⊗ 1 for α ∈ Q. And α ⊗Z 1 = α(1 ⊗ 1) in Q-module
Q ⊗Z Q. Hence Q ⊗Z Q is a 1-dimensional vector space over Q.
2 1 1 1 1 1 1
(Example: Q ⊗Z Q: ⊗Z = · 2 ⊗Z = ⊗Z 2 · = ⊗Z 1.
3 2 3 2 3 2 3
2 1 2 1 2 1 1
However, Q ⊗Q Q: ⊗Q = 1 · ⊗Q = 1 ⊗Q · = 1 ⊗Q .)
3 2 3 2 3 2 3
10.4.5 Let A be a finite abelian group of order n and let pk be the largest power
of the prime p dividing n. Prove that Z/pk Z ⊗Z A is isomorphic to the Sylow p-
subgroup of A.

18
Let P be a Sylow p-subgroup of A with |P | = pk . Define ϕ : Z/pk Z×A → P by (xmodpk Z, a) 7→
xa.

ϕ is bilinear: for r1 , r2 ∈ Z, x1 , x2 ∈ Z/pk Z and a1 , a2 ∈ A


ϕ(r1 x1 +r2 x2 , a) = ϕ(r1 x1 +r2 x2 , a) = (r1 x1 +r2 x2 )a = r1 x1 a+r2 x2 a = r1 ϕ(x1 , a)+r2 ϕ(x2 , a).

ϕ(x1 , r1 a1 + r2 a2 ) = x1 (r1 a1 + r2 a2 ) = x1 r1 a1 + x1 r2 a2 = r1 x1 a1 + r2 x1 a2 = r1 ϕ(x1 , a1 ) +


r2 ϕ(x1 , a2 ).

Thus, ϕ is bilinear. Note that A is subgroup of finite abelian group, hence is abelian. So P is
a Z-module. Since Z/pk Z and A are Z-modules and let Z/pk Z ⊗Z A be the tensor product of
Z/pk Z and A over Z. By Corollary 12 there is a Z-module homomorphism Φ : Z/pk Z⊗Z A → P .

Z/pk Z × A
ι / Z/pk Z ⊗Z A

ϕ Φ
' 
P

Since Φ maps x̄ ⊗ a to xa ∈ P which is an element of order pk . Finally, Z/pk Z ⊗Z A has order


at least pk because pk (x̄ ⊗ a) = pk x̄ ⊗ a = 0 ⊗ a = 0. Hence, Φ is an isomorphism.

10.4.6 If R is any integral domain with quotient field Q, prove that (Q/R)⊗R (Q/R) =
0.

R is an integral domain and Q is the quotient field. So Q = { m


n | m, n ∈ R}. For
m1 , m2 , n1 , n1 ∈ R with ni 6= 0 we have

m1 m2 m1 m2
( mod R) ⊗ ( mod R) = n2 ( mod R) ⊗ ( mod R)
n1 n2 n1 n2 n2
m1 m2
= ( mod R) ⊗ n2 ( mod R)
n1 n2 n2
m1
= ( mod R) ⊗ (m2 mod R)
n1 n2
= 0
since (m2 mod R) = 0.

10.4.11 Let {e1 , e2 } be a basis of V = R2 . Show that the element e1 ⊗ e2 + e2 ⊗ e1 in


V ⊗R V cannot be written as a simple tensor v ⊗ w for any v, w ∈ R2 .

Suppose v = ae1 + be2 , w = ce1 + de2 for a, b, c, d ∈ R. Then


v ⊗ w = (ae1 + be2 ) ⊗ (ce1 + de2 ) = ac(e1 ⊗ e1 ) ⊗ ad(e1 ⊗ e2 ) + bc(e2 ⊗ e1 ) + bd(e2 ⊗ e2 )
If v ⊗ w = e1 ⊗ e2 + e2 ⊗ e1 , then
ac(e1 ⊗ e1 ) ⊗ (ad − 1)(e1 ⊗ e2 ) + (bc − 1)(e2 ⊗ e1 ) + bd(e2 ⊗ e2 ) = 0
Since {e1 ⊗ e1 , e1 ⊗2 , e2 ⊗ e1 , e2 ⊗ e2 } is a basis, we have ac = 0, ad − 1 = 0, bc − 1 = 0 and
bd = 0. Thus, a = 0 and c = 0, but this implies that −1 = 0, a contradiction!

19
10.4.12 Let V be a vector space over the field F and let v, v 0 be nonzero elements
of V . Prove that v ⊗ v 0 = v 0 ⊗ v in V ⊗F V if and only if v = av 0 for some a ∈ F .

Let {v1 , v2 , · · · , vn } be a basis for V . (Here we assume that V is finite dimensional so that for
any element in V , its expression of a FINITE linear combination of the basis makes sense. If
V is infinite, then let’s choose a finite dimensional subspace W ⊂ V containing v and v 0 to
proceed the proof). So V ⊗F V = hvi ⊗ vj | i, j = 1, 2, · · · , ni. Let v, v 0 ∈ V , then v = Σai vi
and v 0 = Σa0i vi . So we have v ⊗ v 0 = (Σai vi ) ⊗ (Σa0i vi ) = Σi Σj (ai a0i )vi ⊗ vj . Similarly, we have
v 0 ⊗ v = Σi Σj (a0i ai )vi ⊗ vj . Hence we have v ⊗ v 0 = v 0 ⊗ v if and only if ai a0j = a0i aj for all i, j.

Now, for fixed j, we have ai a0j = a0i aj for all i if and only if ai = a0 i aj (a0 j )−1 if and only
if ai = (aj (a0 j )−1 )a0 i since F is commutative and every element in F has a multiplica-
tive inverse. Since this is is true for some fixed j, let’s denote the term a = (aj (a0 j )−1 ),
note that a ∈ F . So for this fixed j, we have ai = a · a0 i , i = 1 · · · , n. Hence we have
v = Σai vi = Σ(aa0 i )vi = aΣa0 i vi = av 0 .

Therefore, we proved that v ⊗ v 0 = v 0 ⊗ v if and only if v = av 0 for some a ∈ F .

10.4.15 Show that tensor products do not commute with direct products in general.

N M
Consider Q Z ( Z/2i Z), this implies
i=1
Q ⊗Z ( Z/2i Z) ∼= (Q ⊗Z Z/2Z) ⊕ (Q ⊗Z Z/22 Z) ⊕ (Q ⊗Z Z/23 Z) ⊕ · · ·
L

Since for each i, Z/2i Z is a torsion abelian group (i.e. every element has a finite order), by
exercise 10.4.8(d) (prove later) each term on the right
L handi side is 0, hence the direct product
is 0. However, the left hand side is not zero since Z/2 Z is not a torsion abelian. This is
because there is no integer m such that m( Z/2i Z) = 0 when i goes arbitrarily large. Hence,
L
we conclude that tensor products do not commute with direct products in general.

Now we prove 10.4.8(d): If A is an abelian group group, show that Q ⊗Z A = 0 if and only if
A is a torsion abelian group (i.e. every element of A has finite order).

(⇒)
Suppose Q ⊗Z A = 0. For a ∈ A and q ∈ Q, q = m n for m, n in Z and n 6= 0, we have
0 = q⊗a = m n ⊗ a = m · 1
n ⊗ a = 1
n ⊗ m · a. Since 1
n 6
= 0, so m · a must be zero. So a has finite
order m. Since a ∈ A is arbitrary, so every element in A has finite order hence A is a torsion
group.

(⇐)
Suppose a ∈ A has finite order, say m. i.e. ma = 0 with some m ∈ Z+ . Let q ∈ Q. Then we
q q q
have q ⊗ a = m · m ⊗a= m ⊗m·a= m ⊗ 0 = 0. Hence Q ⊗Z A = 0.

10.5.3 Let P1 and P2 be R-modules. Prove that P1 ⊕ P2 is a projective R-module if


and only if both P1 and P2 are projective.

20
(⇐)
Suppose P1 and P2 are projective modules, by Proposition 30(4), P1 and P2 are direct sum-
mand of free R-modules. i.e. P1 ⊕ Q1 and P2 ⊕ Q2 are free for some modules Q1 and Q2 . But
free module is isomorphic to a direct sum of regular modules, so P1 ⊕ Q1 ⊕ P2 ⊕ Q2 is also
a direct sum of modules, hence is free. Since P1 ⊕ Q1 ⊕ P2 ⊕ Q2 ∼ = (P1 ⊕ P2 ) ⊕ (Q1 ⊕ Q2 ),
therefore, P1 ⊕ P2 is a direct summand of a free module hence is projective.

(⇒)
ϕ
Suppose P1 ⊕ P2 is projective. For R-modules M, N , let M −→ N −→ 0 be an exact sequence,
and f : P1 −→ N an R-module homomorphism. Want to prove that f lifts to an R-module
homomorphism into M . i.e. f lifts to F : P1 −→ M .

Let π : P1 ⊕ P2 −→ P1 be the natural projection (x, y) 7→ x for x ∈ P1 and y ∈ P2 . Then


the composition f ◦ π is an R-module homomorphism. Let nx be the image of x in N . i.e.
f (x) = nx . Since ϕ is surjective (by exactness), so there exists an element mx ∈ M such that
ϕ(mx ) = nx . Now, since P1 ⊕ P2 is a projective R-module, f ◦ π lifts to F 0 : P1 ⊕ P2 −→ M
such that ϕ ◦ F 0 = f ◦ π. i.e. the big diagram commutes and F 0 : (x, y) 7→ mx .

Now, want to prove that f lifts to an R-module homomorphism F : P1 −→ M and the small
diagram commutes. i.e. f = ϕ ◦ F .

Let i1 : P1 → P1 ⊕ P2 be an inclusion and define F = F 0 ◦ i1 : P1 −→ M by sending x to


mx . Certainly F is an R-module homomorphism since F 0 is hence the composition with the
inclusion is. Also we claim that f = ϕ ◦ F . Indeed, for x ∈ P1 , f (x) = nx ∈ N . On the other
hand, ϕ ◦ F (x) = ϕ(F (x)) = ϕ(mx ) = nx . Hence the small diagram commutes and therefore
P1 is a projective R-module.

Similarly we can prove that P2 is a projective R-module by replacing P1 by P2 in the proof.

10.5.7 Let A be a nonzero finite abelian group.


(a) Prove that A is not a projective Z-module.
(b) Prove that A is not a injective Z-module.

(a) By Example (3) on page 391, free Z-modules have no nonzero elements of finite order so
no nonzero finite abelian group can be isomorphic to a submodule of free module. Since A is
abelian, A is not isomorphic to a submodule of a free module. So A is not a direct summand
of a free module. Hence A is not projective.

(b) Since A is a finite abelian group, A ∼


= Zpα1 × · · · × Zpαk . Since each Zpαi is not divisible: for
1 k i
αi
a ∈ Zpαi and for n = pα1 i , there is no x such that xn = a since xpi = 0. Hence Zpα1 ×· · ·×Zpαk
i 1 k
is not divisible. Therefore A is not injective.

10.5.8 Let Q be a nonzero divisible Z-module. Prove that Q is not a projective


Z-module. Deduce that the rational numbers Q is not a projective Z-module.

Lemma If F is a any free module then ∩∞


n=1 nF = 0.

21
Let {x1 , · · · , xk } be a basis for F . Suppose x ∈ ∩∞ n=1 nF . Note that {nx1 , · · · , nxk } be a basis
for nF . So if x ∈ ∩∞ n=1 , x ∈ F , x ∈ 2F , x ∈ 3F, · · · , hence
x = a1 x1 + a2 x2 + · · · + ak xk
= a21 (2x1 ) + · · · + a22 (2x2 ) + · · · + a2k (2xk )
..
.
= an1 (nx1 ) + · · · + an2 (nx2 ) + · · · + ank (nxk )
(Note: the notation aji , here j is an index, not an exponent!)

Thus, 0 = (a1 − nan1 )x1 + · · · + (ak − nank )xk for all k. Hence a1 = nan1 , i.e. a1 = 2a21 =
3a31 = 4a41 = · · · . So n | a1 for all n. Similarly, ak = nank which implies that n | ak for all n.
Therefore, a1 , · · · , ak must be zero, hence x = 0. 

Now, since Q is a divisible Z-module, nQ = Q for all n. Suppose Q is a projective Z-module.


Then Q is a direct summand of a free module F , i.e. Q ⊕ K ∼ = F . So Q is isomorphic to a
submodule of a free module. But ∩∞ n=1 nF = 0 while ∩∞ nQ = ∩∞ Q = Q a contradiction!
n=1 n=1
Therefore, Q is not projective.

10.5.9 Assume R is commutative with 1.


(a) Prove that the tensor product of two free R-modules is free. [Use the fact that
tensor products commute with direct sums.]
(b) Use (a) to prove that the tensor product of two projective R-modules is pro-
jective.

(a) By Corollary 19 in Sec 10.4.


(b) Suppose M, N are projective. Then M ⊕ P = F1 and N ⊕ Q = F2 for some P, Q and free
modules F1 and F2 . Thus,
F1 ⊗R F2 = (M ⊕ P ) ⊗ (N ⊕ Q)
= (M ⊕ P ) ⊗ N ⊕ (M ⊕ P ) ⊗ Q
= (M ⊗R N ) ⊕ (P ⊗ N ) ⊕ (M ⊗ Q) ⊕ (P ⊗ Q)
By (a) F1 ⊗R F2 is free. Thus, M ⊗R N is the direct summand of a free module, hence is
projective.

10.5.15 Let M be a left Z-module and let R be a ring with 1.


(a) Show that HomZ (R, M ) is a left R-module under the action (rϕ)(r0 ) = ϕ(r0 r).
ψ
(b) Suppose that 0 −→ A −→ B is an exact sequence of R-modules. Prove that if
every Z-module homomorphism f from A to M lifts to a Z-module homomorphism
F from B to M with f = F ◦ ψ, then every R-module homomorphism f 0 from A to
HomZ (R, M ) lifts to an R-module homomorphism F 0 from B to HomZ (R, M ) with
f 0 = F 0 ◦ ψ.
(c) Prove that if Q is an injective Z-module then HomZ (R, Q) is an injective R-
module.

(a) For ϕ ∈ HomZ (R, M ), r, r0 ∈ R, define rϕ : R −→ M by rϕ(r0 ) = ϕ(r0 r). First prove that
rϕ ∈ HomZ (R, M ).

22
For r1 , r2 ∈ R, and a ∈ R, we have
- (rϕ)(r1 + r2 ) = ϕ((r1 + r2 )r) = ϕ(r1 r + r2 r) = ϕ(r1 r) + ϕ(r2 r) = rϕ(r1 ) + rϕ(r2 )
- (rϕ)(ar1 ) = ϕ((ar1 )r) = ϕ(a(r1 r)) = aϕ(r1 r) = arϕ(r1 )
by the definition of rϕ and that ϕ is a homomorphism.

Hence rϕ ∈ HomZ (R, M ).

Now, prove that this action of R on HomZ (R, M ) makes it into a left R-module.
Let ϕ, ψ ∈ HomZ (R, M ). First, by Proposition 2 in 10.2, define ϕ + ψ by (ϕ + ψ)(r) =
ϕ(r) + ψ(r), then ϕ + ψ ∈ HomZ (R, M ) and with this operation HomZ (R, M ) is an abelian
group.

Prove that HomZ (R, M ) satisfies the R-module criteria: let r, r0 , s ∈ R,


(1) (r + s)ϕ = rϕ + sϕ
(r + s)ϕ(r0 ) = ϕ(r0 (r + s)) = ϕ(r0 r + r0 s) = ϕ(r0 r) + ϕ(r0 s) = rϕ(r0 ) + sϕ(r0 ) = (rϕ + sϕ)(r0 ).

(2) (rs)ϕ = r(sϕ)


(rs)ϕ(r0 ) = ϕ(r0 (rs)) = ϕ((r0 r)s) = sϕ(r0 r) = r(sϕ(r0 )) since sϕ ∈ HomZ (R, M ).

(3) r(ϕ + ψ) = rϕ + rψ
r(ϕ + ψ)(r0 ) = (ϕ + ψ)(r0 r) = ϕ(r0 r) + ψ(r0 r) = rϕ(r0 ) + rψ(r0 ) = (rϕ + rψ)(r0 )
since ϕ + ψ ∈ HomZ (R, M ).

(4) 1 · ϕ = ϕ
1 · ϕ(r0 ) = ϕ(r0 · 1) = ϕ(r0 ).

Hence, HomZ (R, M ) is a left R-module.

(b) Given an R-module homomorphism f 0 : A −→ HomZ (R, M ) and a ∈ A, define f : A −→


M by a 7→ f 0 (a)(1R ). f is well-defined since f 0 is. Also, f is a Z-module homomorphism since
for a, b ∈ A and r ∈ Z, we have
- f (a + b) = f 0 (a + b)(1R ) = ((f 0 (a) + f 0 (b))(1R ) = f 0 (a)(1R ) + f 0 (b)(1R ) = f (a) + f (b).
- f (ra) = f 0 (ra)(1R ) = rf 0 (a)(1R ) = rf (a).

Now, since ψ is injective, let b ∈ B be the image of a. Since by assumption f lifts to


F : B −→ M and f = F ◦ ψ. So F : b 7→ f (a). Now, define F 0 : B −→ HomZ (R, M )
by F 0 (b)(r) = F (rb). Show that F 0 is an R-module homomorphism.

For b1 , b2 ∈ B, r, s, ∈ R, we have
- F 0 (b1 + b2 )(r) = F (r(b1 + b2 )) = F (rb1 + rb2 ) = F (rb1 ) + F (rb2 ) = F 0 (b1 )(r) + F 0 (b2 )(r)
- F 0 (sb)(r) = F (r(sb)) = F ((rs)b) = F 0 (b)(rs) = sF 0 (b)(r)
by the definition of F 0 and the fact that B is an R-module, F is an R-module. The last equality
holds because F 0 (b) ∈ HomZ (R, M ) hence by (a) we have F 0 (b)(rs) = sF 0 (b)(r).

Hence F 0 is an R-module homomorphism.

Finally, prove that the diagram commutes, i.e. f 0 = F 0 ◦ ψ.


Indeed, f 0 (a)(r) = f 0 (a)(r · 1R ) = rf 0 (a)(1R ) = rf (a) since f 0 (a) is a homomorphism. On the
other hand, (F 0 ◦ ψ)(a)(r) = F 0 (ψ(a))(r) = F 0 (b)(r) = F (rb) = rF (b) = rf (a). Hence, the
23
diagram commutes.

(c) If Q is an injective Z-module, let M = Q, then by (b) any R-module homomorphism


f 0 : A −→ HomZ (R, Q) lifts to an R-module homomorphism F 0 : B −→ HomZ (R, Q) and the
diagram commutes. So by Proposition 34 HomZ (R, Q) is an injective R-module.

10.5.16 Prove that every left R-module M is contained in an injective left R-module.
(a) Show that M is contained in an injective Z-module Q.
(b) Show that HomR (R, M ) ⊆ HomZ (R, M ) ⊆ HomZ (R, Q).
(c) Use the R-module isomorphism M ∼ = HomR (R, M ) and the previous exercise
to conclude that M is contained in an injective R-module.

(a) If M is a Z-module, by Corollary 37, M is contained in an injective Z-module Q.

(b) By (a), we know that M is contained in an injective Z-module Q, let i : M ,→ Q be an


i
inclusion. So we have an exact sequence 0 → M ,→ Q. Apply HomZ (R, −) on this sequence,
by Proposition 27 we have
∗ i
0 −→ HomZ (R, M ) −→ HomZ (R, Q)
which is also exact. Hence, we have HomZ (R, M ) ⊆ HomZ (R, Q).

On the other hand, for ϕ ∈ HomR (R, M ), ϕ : R −→ M is an R-module homomorphism. But


as modules, a fortiori R and M are abelian groups. Note that Z-module homomorphisms are
the same as abelian group homomorphisms, hence ϕ is also an abelian group homomorphism,
hence an Z-module homomorphism. This implies that ϕ ∈ HomZ (R, M ). Hence we have
HomR (R, M ) ⊆ HomZ (R, M ).

Therefore, we proved HomR (R, M ) ⊆ HomZ (R, M ) ⊆ HomZ (R, Q).

(c) If M is an R-module, by exercise 10.5.10(b) we have M ∼


= HomR (R, M ). By (b), we have
M ⊆ HomZ (R, Q). But 10.5.15(c) says that if Q in an injective Z-module, HomZ (R, Q) is an
injective R-module. Hence, we proved that M is contained in an injective R-module.

10.5.25 Prove that A is a flat R-module if and only if for every finitely generated
ideal I of R, the map A ⊗R I → A ⊗R R ∼ = A induced by the inclusion I ⊆ R is again

injective. (or equivalently, A ⊗R I = AI ⊆ A).

10.5.26 Suppose R is a PID. This exercise proves that A is a flat R-module if and
only if A is torsion free R-module (i.e. if a ∈ A is nonzero and r ∈ R, then ra = 0
implies r = 0).
(a) Suppose that A is flat and for fixed r ∈ R consider the map ψr : R → R defined
by multiplication by r : ψr (x) = rx. If r is nonzero show that ψr is an injection.
Conclude from the flatness of A that the map from A to A defined by mapping a
to ra is injective and that A is torsion free.
(b) Suppose that A is torsion free. If I is a nonzero ideal of R, then I = rR for
some nonzero r ∈ R. Show that the map ψr in (a) induces an isomorphism R ∼ =I
ψ η
of R-modules and that composite R −
→I −
→ R of ψr with the inclusion : I ⊆ R is
1⊗ψr 1⊗η
multiplication by r. Prove that the composite A ⊗R R −−−→ A ⊗R I −−→ A ⊗R R
24
corresponds to the map a 7→ ra under the identification A ⊗R R = A and that this
composite is injective since A is torsion free. Show that 1 ⊗ ψr is an isomorphism
and deduce that 1 ⊗ η is injective. Use the preceding exercise to conclude that A
is flat.

Proof

12.1.5 Let R = Z[x] and let M = (2, x) be the ideal generated by 2 and x, considered
as a submodule of R. Show that {2, x} is not a basis of M . [Find a nontrivial
R-linear dependence between these two elements.] Show that the rank of M is 1
but not free rank of 1.

Since x · 2 + (−2) · x = 0 for x, −2 6= 0. Thus {2, x} is not linearly independent, hence is not a
basis of M . Now, prove that the rank of M is 1. Since

M = {2p(x) + xq(x) | p(x), q(x) ∈ Z[x]}



= 2Z[x] + xZ[x]

= 2Z ⊕ xZ[x]

=ϕ Z[x]/(x) ⊕ Z[x]

where ϕ : (2, 0) 7→ (1̄, 0) and (0, x) 7→ (0, 1). Note that Z[x]/(x) is a torsion submodule since
for any 0 6= m ∈ Z[x]/(x), m ∈ Z, we have x · m = 0 because (x) is an ideal. And Z[x] is torsion
free. By exercise 12.1.1(b), rank(M ) = rank(Z[x]) = 1. Hence the rank of M is 1. Finally,
suppose M is free of rank 1, R can be seen as a R-module. Then R/M = Z[x]/(2, x) ∼ = Z2 is
a torsion R-module. By exercise 12.1.1(b), R has rank 1, a contradiction!

12.1.8 Let R be a PID, let B be a torsion R-module and let p be a prime in R.


Prove that if pb = 0 for some nonzero b ∈ B, then Ann(B) ⊆ (p).

For r ∈ Ann(B), then rb = 0 for all b ∈ B. Let (b) be a cyclic submodule generated by b ∈ B,
Ann((b)) ⊆ R is an ideal. Since R is a PID, Ann((b)) = (s) for some s ∈ R. By assumption,
p ∈ Ann((b)) = (s), so p = sr for some r ∈ R. But p is a prime. In PID, prime p is irreducible.
Since s cannot be a unit, thus r is a unit. Hence, s = pr−1 ∈ (p). This implies that (s) ⊆ (p).
Thus, Ann(B) ⊆ Ann((b)) = (s) = (p).

12.1.9 Give an example of an integral domain R and a nonzero torsion R-module


M such that Ann(M ) = 0. Prove that if N is finitely generated torsion R-module
then Ann(N ) 6= 0.

Let R = Z, M = Z/Z ⊕ Z/2Z ⊕ Z/3Z ⊕ · · · . If m ∈ M is nonzero, then m = (a1 , a2 , · · · , an ) for


ai ∈ Z/iZ, i = 1, 2, · · · , n. We see that m ∈ Tor(M ): let x = l.c.m.(1, 2, · · · , n) then xm = 0.
i.e. m is annihilated by l.c.m(1, 2, · · · , n). Now prove Ann(M ) = 0. Let 0 6= m ∈ Ann(M )
with m ∈ Z. But mx 6= 0 for x ∈ Z/(m + 1)Z, a contradiction! Therefore, Ann(M ) must be
zero.

Suppose now N is finitely generated torsion R-module. N = {r1 a1 + · · · + rm am | r1 , · · · , rm ∈


R, a1 , · · · , am ∈ A}
Q for {a1 , · · · , am } ∈ A and for each ai , there exists ri ∈ R such that
ri ai = 0. Let r = ri . Since R is an integral domain, r 6= 0. Hence rN = 0, this implies that
25
Ann(N ) 6= 0.

12.2.3 Prove that two 2 × 2 matrices over F which are not scalar matrices are
similar if and only if they have the same characteristic polynomial.

Let A and B be two 2 × 2 matrices. Suppose A and B are similar, then A = P BP −1 for some
invertible 2 × 2 matrix P . Let λ be an eigenvalue of A. We have
A − λI = P BP −1 − λI = P (B − λI)P −1
Hence
det(A − λI) = det(P (B − λI)P −1 )
= det(P )det(B − λI)det(P −1 )
= det(P )det(P −1 )det(B − λI)
= det(P P −1 )det(B − λI)
= det(B − λI)
Hence A and B have the same characteristic polynomials.

Conversely, if A and B have the same characteristic polynomial p(x). If p(x) is irreducible, then
p(x) is the invariant factor for both A and B. So A, B are similar. Suppoe p(x) = f (x)g(x)
with f, g linear. If f, g have different roots, then p(x) is the minimal polynomial for both
A and B. Then A and B have the same invariant factor, hence they are similar. Now, if
p(x) = (x − a)2 , then the minimal polynomial is f (x) = (x − a). Then A, B are scalar matrices,
a contradiction. Hence, A, B are similar.

12.2.4 Prove that two 3 × 3 matrices are similar if and only if they have the same
characteristic and same minimal polynomials. Give an explicit counterexample to
this assertion for 4 × 4 matrices.

(⇒)
Let A and B be two 3 × 3 matrices. Suppose A and B are similar, then A = P BP −1 for some
invertible 3 × 3 matrix P . Let λ be an eigenvalue of A. We have
A − λI = P BP −1 − λI = P (B − λI)P −1
Hence
det(A − λI) = det(P (B − λI)P −1 )
= det(P )det(B − λI)det(P −1 )
= det(P )det(P −1 )det(B − λI)
= det(P P −1 )det(B − λI)
= det(B − λI)
Hence A and B have the same characteristic polynomials.

(⇐)
Suppose A and B have the same characteristic polynomials p(x). Consider the following cases:

(1) p(x) is irreducible.


Since by Cayley-Hamilton Theorem, the minimal polynomial m(x) divides p(x), so m(x) is
26
irreducible. So p(x) is the only one invariant factor. Then by Theorem, A and B have the
same invariant factors hence are similar.

(2) p(x) = f (x)g(x) = (x − a)(x − b)2 with a, b ∈ F a field and a 6= b.


Since characteristic polynomial p(x) and minimal polynomial m(x) have the same roots, we
have m(x) = (x − a)(x − b) or m(x) = (x − a)(x − b)2 . If m(x) = (x − a)(x − b), since the
characteristic polynomial of A is the product of all the invariant factors of A. So the invariant
factors are
(x − b), (x − a)(x − b)
The rational canonical form is  
a 0 0
0 0 −ab 
0 1 a+b

A and B have the same invariant factors and rational canonical form, hence they are similar.

(3) p(x) = f (x)g(x) = (x − a)(x2 + bx + c) where g(x) is irreducible.


The possible minimal polynomials are m(x) = x − a or m(x) = x2 + bx + c. Suppose
m(x) = x2 + bx + c, but x − a does not divide m(x) so this is not possible. On the other
hand, if m(x) = x − a, but g(x) does not divide m(x). Hence m(x) must be equal to p(x). But
m(x) is the largest invariant factor, so A and B have only one and the same invariant factors.
So they have the same rational canonical form, hence are similar.

(4) p(x) = (x − a)(x − b)(x − c) where a, b, c are distinct.


In this case, m(x) = p(x) since they must have the same roots. As in (3), A and B have the
same invariant factors and rational canonical form hence they are similar.

(5) p(x) = (x − a)3


m(x) can be (x − a)3 , (x − a)2 or (x − a).
If m(x) = (x − a)3 , then p(x) = m(x), so as before A and B are similar.
If m(x) = (x − a)2 , then they have invariant factors
(x − a), (x − a)2
In this case, they have the same rational canonical form:
 
a 0 0
0 0 −a2 
0 1 2a

Hence A and B are similar.

If m(x) = (x − a), A and B are scalar matrices since the invariant factors
(x − a), (x − a), (x − a)
and the rational canonical form:
 
a 0 0
0 a 0
0 0 a
27
Hence A and B are similar.

Example of an 4 × 4 matrix that have the same characteristic polynomial but are not similar.

Let
   
1 0 0 0 0 −1 0 0
0 1 0 0 1 2 0 0 
A= , B= 
0 0 0 −1 0 0 0 −1
0 0 1 2 0 0 1 2

pA (x) = pB (x) = (x − 1)4 and mA (x) = mB (x) = (x − 1)2 . But A and B are not similar since
they do not have the same rational canonical form.

12.2.8 Verify that the characteristic polynomial of the companion matrix


 
0 0 0 · · · 0 −a0
1 0 0 · · · 0 −a1 
 
0 1 0 · · · 0 −a2 
 
 .. .. .. .. .. 
. . . . . 
0 0 0 ··· 1 −an−1

is
xn + an−1 xn−1 + · · · + a1 x + a0 .

We prove by induction. When n = 2, the companion matrix is


 
0 −a0
1 −a1
so we have
 
x −a0
Ix − A =
1 + − a1

So the characteristic polynomial p(x) = det(Ix − A) = x(x + a1 ) + a0 = x2 + a1 x + a0 .

Suppose it is true for n − 1, we have

x 0 ··· 0 a0
 
−1 x · · · 0 a1 
 . .. 
 0 −1 . . . a2
 

 . .. 
 .. . x an−3 
0 0 ··· −1 x + an−2

And p(x) = det(Ix − A) = xk−1 + ak−2 xk−2 + · · · + a1 x + a0 .


When n = k,
28
x 0 ··· 0 a0
 
−1 x · · · 0 a1 
 . .. 
xI − A =  0 −1 . . . a2
 

 . .. 
 .. . x an−2 
0 0 ··· −1 x + an−1 n×n

x 0 ··· 0 a0 x 0 ··· 0
   
−1 x · · · 0 a1 −1 x · · · 0
 . ..   . ..

= x  0 −1 . . . a2  + a0  0 −1 . . .
   

 . ..   . .. 
 .. . x an−3   .. . x
0 0 ··· −1 x + an−2 (n−1)×(n−1) 0 0 ··· −1 (n−1)×(n−1)
So
p(x) = det(xI − A)
= x(xn−1 + ak−1 xk−2 + ak−2 xk−3 + · · · + a2 x + a1 ) + a0 (−1)k−1 · (−1)k+1
= xk + ak−1 xk−1 + · · · + a2 x2 + a1 x + a0
as desired.

12.2.10 Find all similarity classes of 6 × 6 matrices over Q with minimal polynomial
(x + 2)2 (x − 1).

If m(x) = (x + 2)2 (x − 1), consider the following cases of invariant factors:


1. (x + 2), (x + 2), (x + 2), (x + 2)2 (x − 1).
2. (x − 1), (x − 1), (x − 1), (x + 2)2 (x − 1).
3. (x − 1), (x − 1)(x + 2), (x + 2)2 (x − 1).
4. (x + 2), (x − 1)(x + 2), (x + 2)2 (x − 1).
5. (x + 2), (x + 2)2 , (x + 2)2 (x − 1).
6. (x + 2)2 (x − 1), (x + 2)2 (x − 1).

This is because the matrix is 6 × 6, the product of all invariant factors (which is the char-
acteristic polynomial) must have degree 6. Also each invariant factor divides the next. i.e.
a1 (x) | a2 (x) | · · · | an (x). Hence, the rational canonical form are below:
   
−2 1

 −2 

 1



 −2   1 
1.   2.  

 0 0 4  
 0 0 4 
 1 0 0  1 0 0
0 1 −3 0 1 −3
   
1 −2
 0 2   0 2 
   
 1 −1   1 −1 
3.   4.  

 0 0 4  
 0 0 4 
 1 0 0  1 0 0
0 1 −3 0 1 −3
29
   
−2 0 0 4

 0 −4 

1 0 0



 1 −4  0 1 −3 
5.   6.  

 0 0 4

 0 0 4 
 1 0 0  1 0 0
0 1 −3 0 1 −3
12.2.11 Find all similarity classes of 6 × 6 matrices over C with characteristic poly-
nomial (x4 − 1)(x2 − 1).

The characteristic polynomials is p(x) = (x4 −1)(x2 −1) = (x2 +1)(x2 −1)2 = (x2 +1)(x−1)2 (x+
1)2 . The possible m(x) are (x2 + 1)(x − 1)(x + 1), (x2 + 1)(x − 1)2 (x + 1), (x2 + 1)(x − 1)(x + 1)2 ,
(x2 + 1)(x − 1)2 (x + 1)2 . Since p(x) and m(x) have the same roots and m(x) | p(x). Thus, the
corresponding invariant factors are

1. (x − 1)(x + 1), (x2 + 1)(x − 1)(x + 1) = x4 − 1.


2. (x + 1), (x2 + 1)(x − 1)2 (x + 1) = x5 − x4 − x + 1.
3. (x − 1), (x2 + 1)(x − 1)(x + 1)2 = x5 + x4 − x − 1.
4. (x2 + 1)(x − 1)2 (x + 1)2 = x6 − x4 − x2 + 1.

So the rational canonical forms are


   
0 1 −1
1 0




 0 0 0 0 −1 
 0 0 0 1  1 0 0 0 1
1. 
 2. 
 
 1 0 0 0  0 1 0 0 0 
 0 1 0 0  0 0 1 0 0
0 0 1 0 0 0 0 1 1
   
1 0 0 0 0 0 −1
 0 0 0 0 1 1 0 0 0 0 0 
   
 1 0 0 0 1
 4. 0 1 0 0 0 1 
 
3. 
 0 1 0 0 0 0 0 1 0 0 0 
   
 0 0 1 0 0 0 0 0 1 0 1 
0 0 0 1 −1 0 0 0 0 1 0
12.2.15 Determine up to similarity all 2 × 2 rational matrices (i.e. ∈ M2 (Q)) of
precise order 4 (multiplicatively, of course). Do the same if the matrix has entries
from C.

Since the matrix has order 4, i.e. A4 = 1, the minimal polynomial divides x4 − 1 = (x2 +
1)(x + 1)(x − 1). The possibilities for m(x) are
x + 1, x − 1, (x + 1)(x − 1), (x2 + 1), (x2 + 1)(x + 1), (x2 + 1)(x − 1), (x2 + 1)(x + 1)(x − 1).
But the order is precisely 4, so we exclude x + 1, x − 1 and (x + 1)(x − 1). Furthermore, A is
a 2 matrix, degree m(x) ≤ 2. So the only possibilities are (x2 + 1), hence
 
0 −1
A=
1 0
Over C, x2 + 1 = (x − i)(x + i). So the possibilities for m(x) are
x + i, x − i, (x + i)(x − i), (x ± i)(x ± 1).
30
For example, by computation we see that
 
4 i 0
A = =I
0 i
So the 2 × 2 matrix with m(x) = x + i has order 4. Same for other three cases. Therefore, the
corresponding rational canonical forms are
       
i 0 −i 0 0 −1 ±1 0
, , , .
0 i 0 −i 1 0 0 ±i
Note that the second and the third are similar.

12.3.9 Prove that the matrices


   
−8 −10 −1 −3 2 −4
A= 7 9 1 , B =  4 −1 4 
3 2 0 4 −2 5
both have (x − 1)2 (x + 1) as characteristic polynomial but that one can be diagonal-
ized and the other cannot. Determine the Jordan canonical form for both matrices.

By computation, we see that they both have characteristic polynomial p(x) = (x − 1)2 (x + 1).
Since A − I 6= 0 and A + I 6= 0, the minimal polynomial mA (x) must have degree at least two.
But by computation, (A − I)2 6= 0 and (A − I)(A + I) 6= 0. Thus, mA (x) = (x − 1)2 (x + 1)
has no distinct roots, hence A is not diagonalized.

On the other hand, B − I 6= 0 and B + I 6= 0, but (B − I)(B + I) = 0. Thus, mB (x) =


(x − 1)(x + 1) which is linear and has distinct root. Hence B is diagonalizable. Their Jordan
Canonical forms are as follows:
   
1 1 0 1 0 0
JA = 0 1 0  , JB = 0 1 0 
0 0 −1 0 0 −1
12.3.22 Prove that an n × n matrix A with entries from C satisfying A3 = A can be
diagonalized. Is the same statement true over any field F ?

Since A3 = A, the minimal polynomial m(x) divides x3 − x = x(x + 1)(x − 1). Since m(x) has
no repeated roots, A can be diagonalized. However, if charF = 2, take
 
1 1
A=
0 1
It is easy to see that A3 = A but A is not diagonalized.

12.3.23 Suppose A is a 2 × 2 matrix with entries from Q for which A3 = I but


A 6= I. Write A in rational canonical form and in Jordan canonical form viewed as
a matrix over C.

Since A3 = I, so minimal polynomial m(x) divides x√ 3 − 1 which can be factored as (x − 1)(x2 +



x + 1) over Q. But over C, x2 + x + 1 = (x + 2 3i )(x + 1+2 3i ). Since A is a 2 × 2 ma-
1−

trix, so characteristic polynomial


√ is of degree√2 hence minimal polynomial
√ must have a√degree
≤ 2. So m(x) can be (x + 2 ), (x + 2 ), (x − 1)(x + 2 ), (x − 1)(x + 1+2 3i ) or
1− 3i 1+ 3i 1− 3i

31
√ √
(x + 1−2 3i )(x + 1+ 3i
2 ) because A is a 2 × 2 matrix and A 6= I. Hence the possible invariant
factors will be
√ √
1− 3i 1− 3i
(1) (x + 2√ ), (x + 2√ )
1+ 3i 1+ 3i
(2) (x + 2 ), (x + 2 )

(3) (x − 1)(x + 1−2 3i )

(4) (x − 1)(x + 1+2 3i )
√ √
1− 3i 1+ 3i
(5) (x + 2 )(x + 2 )

and the corresponding rational canonical forms are

(1)
" √ #
− 1−2 3i 0√
0 − 1−2 3i

(2)
" √ #
− 1+2 3i 0√
0 − 1+2 3i
√ √ √
1− 3i
(3) Since (x − 1)(x + 2 ) = x2 − ( 1+2 3i )x − 1−2 3i , we have
" √ #
0 1−2√3i
1 1+2 3i
√ √ √
1+ 3i
(4) Since (x − 1)(x + 2 ) = x2 − ( 1−2 3i )x − 1+2 3i , we have
" √ #
0 1+2√3i
1 1−2 3i
√ √
1+ 3i 1− 3i
(5) Since (x − 2 )(x − 2 ) = x2 + x + 1, we have
 
0 −1
1 −1

Now write A in Jordan form.



(1) If m(x) = (x + 1−2 3i ), since the characteristic polynomial p(x) and minimal polynomial
m(x) have

the same roots. And p(x) has degree 2 (since A is a 2 × 2 matrix), so p(x) =
1− 3i 2
(x + 2 ) . Hence we have
" √ #
− 1−2 3i 0√
J=
0 − 1−2 3i
√ √
1+ 3i
(2) If m(x) = (x + 2 ), then p(x) = (x + 1+2 3i )2 . So we have
" √ #
− 1+2 3i 0√
J=
0 − 1+2 3i
32

(3) If m(x) = (x − 1)(x + 1−2 3i ), since minimal polynomial divides characteristic polynomial
and p(x) has degree 2. Hence p(x) = m(x). We have
 
1 0√
J=
0 − 1−2 3i

1+ 3i
(4) If m(x) = (x − 1)(x + 2 ), then p(x) = m(x). We have
 
1 0√
J=
0 − 1+2 3i
√ √
1− 3i 1+ 3i
(5) If m(x) = (x + 2 )(x + 2 ),then p(x) = m(x). We have
" √ #
− 1−2 3i 0√
J=
0 − 1+2 3i

12.3.24 Prove there are no 3 × 3 matrices A over Q with A8 = A but A4 6= I.

Suppose there is a 3 × 3 matrix A over Q with A8 = A. The minimal polynomial divides


x8 − x = (x4 + 1)(x2 + 1)(x + 1)(x − 1). Since A is a 3 × 3 matrix, degree m(x) ≤ 3. Thus, the
possibilities for m(x) are
(x − 1), (x + 1), (x2 + 1), (x − 1)(x + 1), (x2 + 1)(x − 1), (x2 + 1)(x + 1), (x2 + 1)(x − 1)(x + 1)
But by assumption A4 6= I, this implies that m(x) does not divide x4 − 1. Therefore, none of
the above works, hence there are no such matrices exist.

13.1.1 Show that p(x) = x3 + 9x + 6 is irreducible in Q[x]. Let θ be a root of p(x).


Find the inverse of 1 + θ in Q(θ).

Proof

p(x) = x3 + 9x + 6 is irreducible in Q[x] by Eisenstein at p = 3. Let θ be a root of p(x). To


find the inverse of 1 + θ, we can proceed by Euclidean Algorithm in Q[x] as follows:

Note that x3 + 9x + 6 = (x + 1)(x2 − x + 10) − 4 by long division. Rearranging the equation


we have 4 = −(x3 + 9x + 6) + (x + 1)(x2 − x + 10). Now, dividing each term by 4 gives
1 1
1 = − (x3 + 9x + 6) + (x2 − x + 10)(x + 1)
4 4
In the quotient field Q[x]/(x3 + 9x + 6) this equation becomes
1
1 = (x2 − x + 10)(x + 1)
4
1
so that the inverse of 1 + θ in Q(θ) is precisely (θ2 − θ + 10).
4
13.1.5 Suppose α is a rational root of a monic polynomial in Z[x]. Prove that α is
an integer.

Let f (x) = xn + an−1 xn−1 + · · · + a1 x + a0 ∈ Z[x] be a monic polynomial having α as a root.


Want to prove that α is an integer.

33
p
Let α = in lowest term, q 6= 0 and (p, q) = 1. So we have f (α) = 0 which is
q
αn + an−1 αn−1 + · · · + a1 α + a0 = 0
Subtracting a0 from both sides we obtain
αn + an−1 αn−1 + · · · + a1 α = −a0
p
Since α = , so we have
q
pn pn−1 p
+ a n−1 + · · · + a1 = −a0
qn q n−1 q
Factoring out p we get
pn−1 pn−2
 
1
p + an−1 n−1 + · · · + a1 = −a0
qn q q
 n−1 n−2

Since there’s no factor 1
p in the term p qn + an−1 pqn−1 + · · · + a1 1q and a0 is an integer. Hence
p must divide a0 .
p
On the other hand, rearrange f ( ) = 0 in different way we have
q
n pn−1
 
p p
= − an−1 n−1 + · · · + a1 + a0
qn q q
Multiply q n−1 on both sides, we obtain
pn
= − an−1 + an−2 pn−1 q + · · · + a1 pq n−2 + a0 q n−1

q
since the there’s no factor q on the right hand side. Also the right side is actually an integers
since all p, q, ai are. Hence q must be 1.
p p
This proves that α = = = p which is an integer.
q 1
13.1.8 Prove that x5 − ax − 1 ∈ Z[x] is irreducible unless a = 0, 2 or −1. The
first two correspond to linear factors, the third corresponds to the factorization
(x2 − x + 1)(x3 + x2 − 1).

Let f (x) = x5 − ax − 1. First, note that when a = 0, we have f (x) = x5 − 1 = (x − 1)(x4 +


x3 + x2 + x + 1); when a = 2, we have f (x) = x5 − 2x − 1 = (x + 1)(x4 − x3 + x2 − x + 1);
when a = −1, we have f (x) = x5 + x − 1 = (x2 − x + 1)(x3 + x2 − 1).

Now, consider the case where a 6= 0, 2, −1.

Case 1 : If f (x) has at least one linear factor, say x − α, where α ∈ Z.

In this case, we have α5 − aα − 1 = 0, so α5 − aα = 1. Factoring out α we have α(α4 − a) = 1.


So α must divide 1 and since α and a are integers, so α must be ±1. If α = 1, we have
α4 − a = 1 − a = 1 so a = 0. Contradiction! If α = −1, then α4 − a = 1 + a = −1 so a = 2.
Again contradiction!

Case 2 : If f (x) = (x2 + bx + 1)(x3 + dx2 + ex − 1) = 0, i.e. f (x) can be factored into the
product of two irreducible polynomials in Z[x] of degree 2 and 3. (Remark, the product of the
34
constant terms in these two factors is the constant term in f (x) which is −1, so they must be
either 1 or −1).

In this case, we will get a contradiction by doing the following computation.

f (x) = (x2 +bx+1)(x3 +dx2 +ex−1) = x5 +(b+d)x4 +(1+e+bd)x3 +(be+d−1)x2 +(e−b)x−1 =


0, we obtain

b+d=0
1 + e + bd = 0
be + d − 1 = 0

From the first one we have b = −d, then plug into the second and the third we get 1+e−d2 = 0
and −de + d − 1 = 0. The former one gives e = d2 − 1, plugging into the latter one we obtain
d3 − 2d + 1 = 0 which gives (d − 1)(d2 + d − 1) = 0. So d = 1 since d is an integer. Then we
also get e = 0. Now, since a = e − b, so we have a = 0 − 1 = −1. Contradiction!

Case 3 : If f (x) = (x2 + bx − 1)(x3 + dx2 + ex + 1) = 0, similar to Case 2 except swap the
negative sign on the constant terms of the two irreducible factors.

We will do similar computation. So f (x) = (x2 + bx − 1)(x3 + dx2 + ex + 1) = x5 + (b + d)x4 +


(+e + bd − 1)x3 + (be − d + 1)x2 + (b − e)x − 1 = 0, we obtain

b+d=0
e + bd − 1 = 0
be − d + 1 = 0

Similarly, we get b = −d from the first equation, the plug into the second and the third. But
by solving them simultaneously we get d3 + 2d − 1 = 0 which has no integer root since neither
1 nor −1 is a root of this equation (by plugging ±1 into it).

Therefore we proved that this f (x) = x5 − ax − 1 is irreducible over Z[x] except at x = 0, 2, −1.
√ √ √ √ √ √
13.2.7 Prove that Q( 2 + 3) = Q( 2, √ 3). Conclude
√ that [Q( 2 + 3) : Q] = 4. Find
an irreducible polynomial satisfied by 2 + 3.
√ √ √ √
First, prove that Q( 2 + 3) = Q( 2, 3). (⊆) is clear, so we prove (⊇).
√ √ √ √ √ √ √ √ √ √
Denote 2+ 3 by α. So α( 2− 3) = ( 2+ 3)( 2− 3) = −1. This gives 2− 3 = − α1 .
√ √ √ √
Combining this equation with α = 2 + 3 and solving for 2 we get 2 2 = α − α1 , hence
√ √ √ √
2 = 21 (α − α1 ) so 2 ∈ Q(α). Similarly, solving for 3 we get 3 = 21 (α + α1 ), hence
√ √ √ √ √
3√is also
√ in Q(α). √ √So we proved that Q( 2, 3) ⊆ Q(α) = Q( 2 + 3). Therefore,
Q( 2 + 3) = Q( 2, 3).
√ √ √
Now √ that [Q( 2 + 3) : Q] = 4. Consider the extensions of fields Q ⊆ Q( 2) ⊆
√ prove
Q( 2, 3).

35
√ √
Q( 2)/Q is degree 2 since the minimal √ polynomial for 2√ over Q is x2 − 2 (monic, irreducible
by √ Eisenstein
√ at
√ p = 2 and having 2 as a root) so [Q( 2) :2 Q] = 2. On the other hand, √
Q( 2, 3)/Q( 2) has degree √ at most
√ 2 and is 2 if and √ only if x −√3 is irreducible over Q( 2).
It remains to prove√that 3 6∈ Q( 2). If it √ were, then 3 = a + b 2 for a, b ∈ Q. So we have
2 2
3 = a + 2b + 2ab 2. If a, b 6= 0, solve for 2 we get
√ 3 − a2 − 2b2
2=
2ab

a√ contradiction
√ since this implies that √ 2 ∈ Q which is not√ true. Now, if a = 0 we have
3 = b 2, multiplying both side
√ by 2 we found that 6 is a rational, a contradiction.
If
√ b = 0,√we would have that 3 = a a rational, again a contradiction. This proved that
3 6∈ Q( 2).
√ √ √
Hence Q( 2, 3)/Q( 2) √ has
√ degree 2, then by Theorem 14, the multiplication
√ √ rule √ for fields

extensions,√we have
√ [Q( 2, 3) : Q] = 4. But we just proved that Q( 2 + 3) = Q( 2, 3).
Hence [Q( 2 + 3) : Q] = 4, as desired.
√ √
Finally, find an irreducible polynomial satisfied 2+ 3.
√ √ √ √
Let α = 2 + 3. Square both sides we get α2 = 5 + 2 6, so α2 − 5 = 2 6. Square both
sides again we have α4 − 10α√
2 + 25 = 24 which is α4 − 10α2 + 1 = 0. Hence the polynomial

x4 − 10x2 + 1 satisfies α
√ √ = 2 + 3. This polynomial is monic and
√ has
√ α as a root. Since we
just proved that [Q( 2, 3) : Q] = 4 and by Proposition 11, [Q( 2, 3) : Q] = deg mα (x).
This implies that the minimal polynomial for α has degree 4 so it is actually the polynomial
x4 − 10x2 + 1. This implies that x4 − 10x2 + 1 is irreducible.

13.2.8 Let F be a field of characteristic 6= 2. Let


√ D√ 1 and D2 be elements of F ,
neither of which is a square in F . Prove that F ( D1 , D2 ) is of degree 4 over F if
D1 D2 is not a square in F and is of degree 2 over F otherwise.

The√ proof
√ is basically similar to 13.2.7. Consider the field extensions F ⊆ F ( D1 ) ⊆
F ( D1 , D2 ).

First, show that [F ( D1 ) : F ] = 2.

Since D1 is not
√ a square, so D1 6∈ F . And D1 is a root of x2 − D1 . Since the other root of
x2 − D1 is − √ D1 and neither of them√is in F , so x2 − D1 is irreducible hence is the minimal
polynomial of D1 over F . Hence [F ( D1 ) : F ] = deg m√D1 (x) = 2.
√ √ √ 2
Note that √ [F ( D1 , D2 ) : F ( D1 )] is at most 2√and is 2 if√and only if√x − D2 is√irreducible
over F ( D1 ),√and this is true if and only if D2 6∈ F ( D1 ). If D2 ∈ F ( D√
√ 1 ), then
D2 = a + b D1√for a, b ∈ F . Squaring both sides we get D2 = a2 + b2 D1 + 2ab D1 . If
a, b 6= 0, solve for D1 we get
p a2 + D1 b2
D1 = ∈F
2ab
which is a contradiction since D1 is not a square in F .

If b = 0, we have a ∈ F , a contradiction.
√ D2 =√ √ √
If a = 0, then D2 = b D1 . Multiplying both sides by D2 we get D2 = b D1 D2 . This
36
√ √ √ √ √
√ that D2 ∈ F ( D1 ) if D1 D2 is a square in F . Hence [F ( D1 , D2 ) : F ] = [F ( D1 ) :
implies
F ( D1 )] = 2 if D1 D2 is a square in F .
√ √ √ √
If D
√1 D2 is not a square in
√ F , from
√ above we know √ that
√ D2 6∈√F ( D1 ).√So [F ( D1 , D2 ) :
F ( D1 )] = 2. Hence [F ( D1 , D2 ) : F ] = [F ( D1 , D2 ) : F ( D1 )][F ( D1 ) : F ] = 2·2 = 4.

13.2.13
√ Suppose F = Q(α1 , α2 , · · · , αn ) where αi2 ∈ Q for i = 1, 2, ·, n. Prove that
3
2 6∈ F .

Note that each αi satisfies the polynomial x2 − αi2 ∈ Q[x]. So


[Q(α1 , α2 , · · · , αi+1 ) : Q(α1 , α2 , · · · , αi )] ≤ 2.
√ √ √
Thus, [F : Q] = 2r for some r ≤ n. Suppose√3 2 ∈ F , then Q( 3 2) ⊆ F . But [Q( 3 2) : Q] = 3.
The degrees of the fields extensions Q ⊆ Q( 3 2) ⊆ F imply that 3 | 2r , a contradiction!

13.2.14 Prove that if [F (α) : F ] is odd then F (α) = F (α2 ).

Consider the fields extension F ⊆ F (α2 ) ⊆ F (α). Since α satisfies the polynomial x2 − α2 over
F (α2 ) which has degree 2. This implies that [F (α) : F (α2 )] ≤ 2. Assume that [F (α) : F ] is
odd, we must have [F (α) : F (α2 )] = 1. i.e. F (α) = F (α2 ).

13.2.17 Let f (x) be an irreducible polynomial of degree n over a field F . Let g(x)
be any polynomial in F [x]. Prove that every irreducible factor of the composite
polynomial f (g(x)) has degree divisible by n.

Without loss of generality we assume that f (x) is monic (if not, we could divide f (x) by its
leading coefficient to obtain a monic polynomial). Let p(x) be an irreducible factor of f (g(x)),
so p(x) divides f (g(x)). So p(x) is monic and has degree k ≤ n. Let α be a root of p(x), we
have [F (α) : F ] = deg minα (x) = k.

Since α is a root of p(x), so α is also a root of f (g(x)). So f (g(α)) = 0 and this implies that g(α)
is a root of f (x). So f (x) is the minimal polynomial for g(α). Hence we have [F (g(α)) : F ] =
deg ming(α) (x) = n. Now consider the field extensions

F ⊆ F (g(α)) ⊆ F (α)
By the multiplication rule of fields extension, we see that [F (g(α)) : F ] divides [F (α) : F ]. i.e.
n | k.

13.2.18 Let k be a field and let k(x) be the field of rational functions in x with
P (x)
coefficients from k. Let t ∈ k(x) be the rational function with relatively prime
Q(x)
polynomials P (x), Q(x) ∈ k[x], with Q(x) 6= 0.
(a) Show that the polynomial P (X) − tQ(X) in the variable X and coefficients in
k(t) is irreducible over k(t) and has x as a root.
(b) Show that the degree of P (X) − tQ(X) as a polynomial in X with coefficients
in k(t) is the maximum of the degrees of P (x) and Q(x).
P (x)
(c) Show that [k[x] : k(t)] = [k(x) : k( )] = max (deg P (x), deg Q(x)).
Q(x)
37
(a) First, P (X) − tQ(X) is irreducible over (k[X])[t] since it is nothing but a polynomial of
degree one in t. But because (k[t])[X] = k[t, X] = k[X, t] = (k[X])[t], and P (X), Q(X) are
relatively prime. So P (X) − tQ(X) is irreducible over (k[t])[X]. (If not, P (X) − tQ(X) =
F (x, t)G(X), note that F and G can’t be both in t since their product has degree one in t.
But this implies that G(X) divides both P (X) and Q(X), contradiction!). Now, since k[t] is a
UFD, by Gauss Lemma P (X)−tQ(X) is irreducible over its field of fractions which is (k(t))[x].
P (X)
Furthermore, if we plug x into P (X) − tQ(X) we get P (x) − tQ(x). But t = Q(X) , so we have
P (X)
P (x) − tQ(x) = P (x) − Q(X) Q(x) = 0 hence P (X) − tQ(X) has x as a root.

(b) Let P (x) = xn + an−1 xn−1 + · · · + a1 x + a0 and Q(x) = xm + bm−1 xm−1 + · · · + b1 x + b0


(without loss of generality, we assume P (X) and Q(X) are monic, if not, we could divide each
of them by its leading coefficient to obtain a monic polynomial.) We assume that n > m > 0,
then we have

P (X) − tQ(X) = (xn + an−1 xn−1 + · · · + a1 x + a0 ) − t(xm + bm−1 xm−1 + · · · + b1 x + b0 )


= (xn + an−1 xn−1 + · · · + a1 x + a0 ) − txm − tbm−1 xm−1 − · · · − tb1 x − tb0
= xn + · · · + (am − tbm )xm + · · · + (a1 − tb1 )x + (a0 − tb0 )

Hence degree of P (X) − tQ(X) is n which is the maximum of the degree of P (x) and Q(x).
P (x)
(c) The first equality is clear since t = Q(x) by assumption. On the other hand, in part (a) we
assumed that the polynomial P (X) − tQ(X) is monic, and proved that it is irreducible and
has x as a root hence is the minimal polynomial of x over k(t). Also part (b) proved that it
has degree n. Hence we have [k(x) : k(t)] = deg minx (X) = max (deg P (x), deg Q(x)) = n.

13.2.22 Let K1 and K2 be two finite extensions of a field F contained in the field K.
Prove that the F -algebra K1 ⊗F K2 is a field if and only if [K1 K2 : F ] = [K1 : F ][K2 : F ].

Let {αi } be a basis for K1 over F and {βj } be a basis for K2 over F . Then {αi ⊗ βj } is a
basis for K1 ⊗F K2 over F . Define a map ϕ : K1 ⊗F K2 → K1 K2 by ϕ(αi ⊗ βj ) = αi βj , and
extend it by linearity. It is easy to check that ϕ is an F -algebra homomorphism. The map ϕ
is surjective because the elements αi βj span K1 K2 as an F -vector space.

(⇐) By assumption the F -vectors spaces K1 ⊗F K2 and K1 K2 have the same dimension over
F , namely [K1 : F ][K2 : F ]. Thus by linear algebra ϕ is injective and hence an isomorphism.
Therefore, K1 ⊗F K2 is isomorphic to the field K1 K2 .

(⇒) Conversely, if K1 ⊗F K2 is a field then it has no non-zero ideals so ker(ϕ) = 0. Therefore


ϕ is an isomorphism and [K1 K2 : F ] = [K1 ⊗F K2 : F ] = [K1 : F ][K2 : F ].

13.4.1 Determine the splitting filed and its degree over Q for x4 − 2.
√ √ √ √
Since
√ f (x)
√ = x4 − 2 can be factored as (x − i 4 2)(x + i 4 2)(x − 4 2)(x + 4 2), the roots are
± 4 2, ±i 4 2.

First, prove that the splitting field for f (x) is Q( 4 2, i).
Let K be the splitting field for f (x). By definition of the splitting field, K contains all roots of
38
√ √ √ √
f (x) hence contains 4 2 and
√ the ratio
√ of two roots i 4 2, 4 2 which
√ is i. So K ⊇ Q( 4 2, i).
√ On
all roots ± 4 2, ±i 4 2 clearly lie in the field Q( 4 2, i), so we have K ⊆ Q( 4 2, i).
the other hand, √
Hence, K = Q( 4 2, i).

Now, prove that [Q( 4 2, i) : Q] = 8
√ √
Consider the extensions of fields: Q ⊆ Q( 4 2) ⊆ Q( 4 2, i).

(1) The extension Q( 4 2)/Q has degree 4 since it satisfies the polynomial x4 − 2 which
√ is ir-
4 4
√ at p = 2. So x − 2 is the minimal polynomial for 2 over Q.
reducible over Q by Eisenstein
And it is monic. Hence [Q( 4 2) : Q] = deg min √
4 (x) = 4.
2
√ √
(2) The extension Q( 4 2, i)/Q( 4 2) has degree at most 2 since it satisfies the polynomial x2 + 1
and is precisely 2 if and only
√ if x2 + 1 is irreducible.
√ Indeed,
√ x2 + 1 is irreducible over Q (none
4 4 4
of its roots i, −i are in Q( 2). So we have [Q( 2, i) : Q( 2)] = 2.

Hence, by multiplication rule for field extensions, we have



4

4

4

4
[Q( 2, i) : Q)] = [Q( 2, i) : Q( 2)][Q( 2) : Q)] = 4 · 2 = 8

i.e. [Q( 4 2, i) : Q)] has degree 8.

13.4.2 Determine the splitting filed and its degree over Q for x4 + 2.

First, find the roots of this polynomial. If α is a root of this equation, then α4 = −2, then
(ξα)4 = −2 where ξ is any 4th root of −1. Hence the solutions of this equation are

ξ a 4th root of − 1
4
ξ 2,
π
To compute√ξ explicitly,

since x4 + 1 = (x2 + i)(x2 − i), so x2 = i = e 2 i hence x =
π π
±e 4 i = ±( 22 + 22 i). On the other hand, we have x2 = −i = e− 2 i which implies that
π
√ √ √ √
x = ±e− 4 i = ±( 2
2√ − 2
2 i). Hence ξ = ± 2
2
± 2
2 i. The roots of the polynomial x4 + 2 are

2 2

4
therefore (± 2 ± 2 i) 2.

Prove that the splitting field for the polynomial x4 + 2 is Q( 4 2, i).

Let K be the
√ splitting field for √x4 +√2 over Q. K must√ contains all roots of this√ polynomial.
√ √ √ √ √ √ √
Let α1 = ( 2 + 2 i) 2, α2 = ( 2 − 2 i) 2, α3 = (− 2 + 2 i) 2 and α4 = (− 22 − 22 i) 4 2.
2 2 4 2 2 4 2 2 4

√ α1 α4 3 √
Hence 4 2 = (α1 + α4 )/(α1 α4 ) and i = (α1 + α3 )( ) . So K contains 4 2 and i. i.e.
√ α1 + α4
K ⊇ Q( 4 2, i).
√ √ √ √
On the other hand, all roots lie in the field Q( 4 2, i). For example, 4 2( 22 + 22 i) is a product
√ √ √ √ √ √
of 2 4 and 22 + 22 i, but the latter is just a linear combination of 2 and i, where 2 = ( 4 2)2 .
√ √ √ √
Hence the root 4 2( 22 + 22 i) lies in the field Q( 4 2, i). Similarly all other roots also lie in the
√ √ √
4
field Q( √ 2, i), so K ⊆ Q( 4 2, i). Hence, the splitting field for x4 + 2 over Q is Q( 4 2, i). i.e.
K = Q( 4 2, i).

Now, compute the degree extension. The degree is actually the same as the exercise 13.4.1
since the polynomials x4 − 2 and x4 + 2 have the same splitting field over Q. So I will just do
39
the copy and paste:
√ √
Consider the extensions of fields: Q ⊆ Q( 4 2) ⊆ Q( 4 2, i).

(1) The extension Q( 4 2)/Q has degree 4 since it satisfies the polynomial x4 − 2 which
√ is
irreducible over Q by Eisenstein at p = 2 (Remark: although we just proved that Q( 4 2, i) is
the splitting field for the polynomial x4 + 2. To compute the degree
√ of extension, we should
know that the minimal polynomial would be x4 − 2 over√Q since 4 2 is a root of x4 − 2 but
not √of x4 + 2). So x4 − 2 is the minimal polynomial for 4 2 over Q. And it is monic. Hence
4
[Q( 2) : Q] = deg min √ 4 (x) = 4.
2
√ √
(2) The extension Q( 4 2, i)/Q( 4 2) has degree at most 2 since it satisfies the polynomial x2 + 1
and is precisely 2 if and only if x2 + 1 is irreducible.
√ Indeed,
√ x2 + 1 is irreducible over Q (none
4 4
of its roots i, −i are in Q). So we have [Q( 2, i) : Q( 2)] = 2.

Hence, by multiplication rule for field extensions, we have



4

4

4

4
[Q( 2, i) : Q)] = [Q( 2, i) : Q( 2)][Q( 2) : Q)] = 4 · 2 = 8

i.e. [Q( 4 2, i) : Q)] has degree 8.

13.5.3 Prove that d divides n if and only if xd − 1 divides xn − 1.

If d divides n, then n = dq for some integer q. So we have xn − 1 = xdq − 1 = (xd − 1)(xq−1 +


xq−2 + · · · + x + 1). Hence xd − 1 divides xn − 1.

On the other hand, let n = dq + r we have

xn − 1 = (xdq+r − xr ) + (xr − 1) = xr (xdq − 1) + (xr − 1)

So if xd − 1 divides xn − 1, since xd − 1 also divides xdq − 1 = (xd − 1)(xq−1 + xq−2 + · + 1).


Thus xd −1 must also divides xr −1. But r < d, r must be 0. Therefore, n = dq. i.e. d divides n.

13.5.4 Let a > 1 be an integer. Prove for any positive integers n, d that d divides
n if and only if ad − 1 divides an − 1. Conclude in particular that Fpd ⊆ Fpn if and
only if d divides n.

By plugging a into previous exercise we get d divides n if and only if ad − 1 divides an − 1.

To prove that Fpd ⊆ Fpn if and only if d divides n.


d
Note that Fpd is the field consisting of the roots of xp − x over Fp . Similarly Fpn is the
n
field consisting of the roots of xp − x over Fp . If d divides n, by previous exercise we have
d n d
pd − 1 divides pn − 1, this implies that xp −1 − 1 divides xp −1 − 1. Hence xp − x divides
n d n
xp −x which means that the roots of xp −x is contained in the roots of xp −x. i.e. Fpd ⊆ Fpn .
d n
On the other hand, if Fpd ⊆ Fpn . Then the roots of xp − x is contained in the roots of xp − x.
d n d n
Hence we have xp − x divides xp − x which implies that xp −1 − 1 divides xp −1 − 1. By
previous exercise, pd − 1 divides pn − 1, hence d divides n.

40
13.5.7 Suppose K is field of characteristic p which is not a perfect field: K 6= K p .
Prove there exist irreducible inseparable polynomials over K. Conclude that there
exist inseparable finite extensions of K.

Suppose K 6= K p , then there is an element α in K with α 6= β p for every β ∈ K. So


f (x) = xp − α has no root in K. Hence f (x) is irreducible over K. Moreover, f (x) is insep-
arable since its derivative f 0 (x) = pxp−1 = 0 is relatively prime to f (x). Therefore, f (x) is
inseparable by Corollary 36 in 13.5.

13.5.8 Prove that f (x)p = f (xp ) for any polynomial f (x) ∈ Fp [x].

By Proposition 35, for any a1 , a2 ∈ Fp , (a1 + a2 )p = ap1 + ap2 . Suppose (a1 + · · · + an−1 )p =
ap1 +· · ·+apn−1 , by induction we have (a1 +· · ·+an−1 +an )p = (a1 +· · ·+an−1 )p +apn = ap1 +· · ·+apn .

Moreover, by Fermat’s Little Theorem ap ≡ a (mod p) for a ∈ Z we have


f (x)p = (xn + an−1 xn−1 + · · · + a1 x + a0 )p
= xnp + apn−1 xp(n−1) + · · · + ap1 xp + ap0
= (xp )n + an−1 (xp )n−1 + · · · + a1 (xp ) + a0
= f (xp )
Hence f (x)p = f (xp ) for any polynomial f (x) ∈ Fp [x].

13.6.15 Let p be an odd prime not dividing m and let Φm (x) be the mth cyclotomic
polynomial. Suppose a ∈ Z satisfies Φm (a) ≡ 0 (mod p). Prove that a is relatively
prime to p and that the order of a in (Z/pZ)× is precisely m.
Y Y
Note that xm − 1 = Φd (x) and can also be written as Φm (x) · Φd (x). Let p be an
d|m d|m,d<m
odd prime and p - n. Suppose a ∈ Z satisfies Φm (a) ≡ 0 (mod p), then p divides Φm (a). Hence
p divides am − 1. i.e am − 1 ≡ 0 (mod p) or am ≡ 1 (mod p). Thus a is relatively prime to
p (if not, then a = pb for some integer b, then we would have bm pm ≡ 0 (mod p), contradiction.).

To compute the order of a ∈ Z/pZ, note that the order of a (mod p) must be less than or
equal to m from what we just proved. Suppose a has order less than m, then al ≡ 1 (mod p)
for some Qinteger l with l < m and l | m. This implies that al − 1 ≡ 0 (mod p). So we have
l
a − 1 = d|l Φd (a) ≡ 0 (mod p). This implies that Φk (a) ≡ 0 (mod p) for some integer k ≤ l
and k | l. Now, Φk (x) has a as a root (mod p) with k ≤ l < m and k | m. This implies that
xm − 1 would have a as a multiple root mod p. Since we know that xm − 1 is separable which
means it has distinct roots (example (2) on page 547). Contradiction! Hence the order of a
(mod p) is preciesly m.

13.6.16 Let a ∈ Z. Show that if p is an odd prime dividing Φm (a) then either p
divides m or p ≡ 1 (mod m).

Suppose p - m. Since p divides Φm (a), by 13.6.15 the order of a (mod p) is m. i.e. am ≡ 1


(mod p). But a ∈ (Z/pZ)× , so m divides |Z/pZ| = p − 1 by Lagrange. Hence p − 1 ≡ 0 (mod
m) or p ≡ 1 (mod m).

41
Hence we proved that either p divides m or p ≡ 1 (mod m).

13.6.17 Prove there are infinitely many primes p with p ≡ 1 (mod p).

We will prove by assuming 13.6.14. Consider Φm (1), Φm (2), · · · , Φm (n), · · · ∈ Z[x]. These are
monic polynomial in Z[x] of degree at least one, so by 13.6.14 there are infinitely many distinct
primes dividing Φm (i) for i = 1, 2, · · · . But in 13.6.16 we just proved that either p divides m or
p ≡ 1 (mod m). Since only finitely many of them are divisors of m because m is finite, hence
there are infinitely many primes p with p ≡ 1 (mod m).
√ √
14.1.4 Prove that Q( 2) and Q( 3) are not isomorphic.
√ √
Suppose Q( 2) and Q( 3) are isomorphic. Want to draw a contradiction.
√ √ √ √
√ 2) →
Let√σ : Q( √Q(√ 3) be an isomorphism, since σ√is a√homomorphism, we have σ( 2· 2) =
σ( 2)σ( 2) = 3 3 = 3. On the other hand, σ( 2 · 2) = σ(2) = σ(1) + σ(1) = 1 + 1 = 2
by the fact that an homorphism maps the identity to identity. Hence we obtain 2 = 3, a
contradiction!

Here is alternative
√ less-technical
√ proof:
Let σ : Q( 2) −→ Q( 3) be an isomorphism. Then σ takes 1 to 1 i.e. σ(1) = 1, it follows
that σ(a) = a for a in the prime subfield Q. Hence σ fixes the prime subfield Q.
√ √
Now, since Q( 2) is the field generated by√ 2 over Q. √And the isomorphism
√ σ fixes
√ Q, so σ
is completely determined by its action on
√ 2, i.e. by σ( 2). So let σ( 2) = a + b 3 for some
a, b ∈ Q. The minimal polynomial for√ 2 over Q is x2 − 2 (since
√ it is monic, irreducible by
Eisenstein Criterion at p = 2 and has 2 as a root). Then σ( 2) must also be a root of this
polynomial
√ (any automorphism
√ permutes the roots of an irreducible polynomial). However,
σ( 2) of the form a + b 3 cannot be its root.
√ √
If it were, then (a + b 3)2 − 2 = a2 + √
2ab 3 + 3b2 − 2 = 0. If a = b = 0, we have −2 = 0,
contradiction (or, in this case, σ maps 2 toqzero which means that σ is a zero map, contra-
diction). If a = 0, b 6= 0, 3b2 = 2, then b = 23 ∈
6 Q, contradiction. If a 6= 0, b = 0, then we
√ √ 2 −3b2
have a = 2, a contradiction. Finally if a 6= 0, b =6 0, then we have 3 = 2−a2ab , again a

contradiction since 3 is not rational.
√ √ √
Hence
√ σ( 2) cannot be of the form of a + b 3 which implies that Q( 2) is not isomorphic to
Q( 3).
√ √
14.1.5 Determine the automorphisms of the extension Q( 4 2)/Q( 2) explicitly.
√ √ √ √ √
Note that 4 2 is satisfies the polynomial x2 √
− 2 over Q( 2) so the [Q(√
4
2) : Q( √2) has degree

at most 2 but is precisely 2 since the root 4 2 is not contained
√ in√Q( 2). So Q( 4 2)/Q( 2) is
a quadratic extension.
√ Any automorphism
√ √ σ ∈ Aut(Q(√4 2)/Q( 2) is completely determined
by its action on 4 2. i.e. σ sends 4 2 to − 4 2 fixing Q( 2).

42
14.1.7 Determine Aut(R/Q).
(a) Prove that any σ ∈ Aut (R/Q) takes squares to squares and takes positive reals
to positive reals. Conclude that a < b implies σa < σb for every a, b ∈ R.
1 1 1 1
(b) Prove that − m < a−b < m implies − m < σa − σb < m for every positive integer
m. Conclude that σ is a continuous map on R.
(c) Prove that any continuous map on R which is the identity on Q is the identity
map, hence Aut(R/Q) = 1.

(a) For α2 ∈ R, we have σ(α2 ) = σ(α · α) = σ(α)σ(α) = (σ(α))2 ∈ R since σ is an auto-


morphism. On the other hand, for α ∈ R and α > 0, since α is a real number, α = β 2
for some β ∈ R and β 6= 0. So we have σ(α) = α(β 2 ) = (σ(β))2 > 0 (since β 6= 0, so
σ(β) 6= 0 because σ is an automorphism hence injective). Thus σ ∈ Aut (R/Q) takes squares
to squares and takes positive reals to positive reals. Now if a < b for a, b ∈ R, then b−a > 0, we
have σ(b−a) = σ(b)−σ(a) > 0 since σ takes positive reals to positive reals. Hence σ(b) > σ(a).

(b) Since any automorphism σ ∈ Aut(R/Q) fixes Q. For every positive integer m, we have
1 1 1 1
− = σ(− ) < σ(a − b) = σ(a) − σ(b) < σ( ) =
m m m m
Now, we conclude that σ is continuous on R. Given any  > 0, for any positive integer m and
1
let δ = m < . If | a − b |< δ, we have
1
| σ(a) − σ(b) |< <
m
since  is arbitrary, hence we conclude that σ is a continuous map on R for every a, b ∈ R.

(c) In real analysis we know that for any real number a ∈ R, there exists a sequence of rational
numbers {qn } such that {qn } → a as n → ∞ (Q is dense in R). Now, let σ be any continuous
map on R which is the identity on Q. We have
a = lim qn = lim σ(qn ) = σ( lim qn ) = σ(a)
n→∞ n→∞ n→∞

since σ fixes Q and note that limit can exchanges with the continuous function σ.

Hence σ(a) = a. i.e. Any automorphism from R to R fixing Q must be the identity.

14.1.8 Prove that the automorphisms of the rational function field k(t) which fix
at + b
k are precisely the fractional linear transformations determined by t 7→ for
ct + d
a, b, c, d ∈ k, ad − bc 6= 0.

Let σ : k(t) −→ k(t) be an automorphism fixing the field k. Since σ is completely deter-
p(t)
mined by its action on t, let σ : t 7→ for polynomials p(t), q(t) in k[t] with q(t) 6= 0. i.e.
q(t)
p(t)
σ(t) = . Want to prove that p(t) and q(t) are both linear.
q(t)
Now, want to use exercise the result of 13.2.18 in previous homework, by adapting suitable
variables in this exercise. We have
p(t)
[k(t) : k(σ(t))] = [k(t) : k( )] = max (deg p(t), deg q(t))
q(t)
43
p(t) p(t)
As proved in exercise 13.2.18, k(t) is an extension of k( ). So k( ) ⊆ k(t). But σ is
q(t) q(t)
p(t) p(t)
an automorphism mapping t to , hence [k(t) : k( )] must be 1. This implies that max
q(t) q(t)
(deg p(t), deg q(t)) is 1 which means that both p(t) and q(t) has degree no greater than one.
That is, p(t) = at + b, q(t) = ct + d for a, b, c, d ∈ k and p(t), q(t) are relatively prime, i.e.
bc at + b
ad 6= bc (If ad − bc = 0, then ad = bc, rearrange we have a = , plugging into we obtain
d ct + d
bc b
dt+b (ct + d) b
= d = which means at + b and ct + d are not relatively prime). Therefore,
ct + d ct + d d
at + b
the automorphism σ is determined by t 7→ for a, b, c, d ∈ k and ad − bc 6= 0.
ct + d

at + b
On the other hand, if ad − bc 6= 0, then σ(t) = defines an automorphism. Clearly σ
ct + d
p(t)
is well-defined since it is independent of choice of t. And it maps a rational function to
q(t)
p( at+b
ct+d )
a rational function at+b ∈ k(t) by just simplifying by doing multiplication on the fraction
q( ct+d )
dt − b
and use common denominator ct + d. The inverse is given by σ −1 (t) = , by taking the
−ct + a
inverse on the matrix  
a b
c d
An easy verification can be done as below:
at + b d( at+b
ct+d ) − b
adt+bd−bct−bd
ct+d t(ad − bc)
σ −1 σ(t) = σ −1 ( )= = −act−bc+act+ad
= =t
ct + d −c( at+b
ct+d ) + a ct+d
ad − bc
Similarly, we have
dt − b a( dt−b ) + b adt−ab−bct+ab
t(ad − bc)
σσ −1 (t) = σ( ) = −ct+a
dt−b
= −ct+a
cdt−bc−cdt+ad
= =t
−ct + a c( −ct+a )+d −ct−a
ad − bc
Hence σ is a bijection. Also, it is a homomorphism since for f (t), g(t) ∈ k(t), we have
at + b at + b at + b
σ(f + g)(t) = (f + g)( ) = f( ) + g( ) = σ(f (t)) + σ(g(t))
ct + d ct + d ct + d
at + b at + b at + b
σ(f g)(t) = (f g)( ) = f( )g( ) = σ(f (t))σ(g(t))
ct + d ct + d ct + d
at + b
Therefore σ(t) = for ad − bc 6= 0 defines an automorphism on k(t).
ct + d
14.2.3 Determine the Galois group of (x2 − 2)(x2 − 3)(x2 − 5). Determine all the
subfields of the splitting field of this polynomial.
√ √ √
The extension K = Q( 2, 3, 5) is Galois over Q since it is the splitting field of the polyno-
mial (x2 − 2)(x2 − 3)(x5 − 2) since all roots are in K. On the other hand, any field containing
these roots√contains
√ K.√Any automorphism σ is completely√ √determined
√ by its action on the
generators 2, 3 and 5 which must be mapped to ± 2, ± 3 and ± 5 respectively. Hence
44
the only possibilities for automorphisms are the maps
 √ √  √ √  √ √
 √2 → √ − 2  √2 → √ 2  √2 → √2
σ: 3 → √3 τ: 3→√ − 3 δ: 3→ √ 3
 √  √  √
5→ 5 5→ 5 5→− 5
It’s clearly to see that σ 2 = τ 2 = δ 2 = 1. Moreover,
 √ √  √ √  √ √
 √2 → −√2  √2 → √− 2  √2 → √ 2
στ = τ σ: 3→√ − 3 σδ = δσ: 3→ √ 3 τ δ = δτ : 3 → −√3
 √  √  √
5→ 5 5→− 5 5→− 5
That is, three maps commute with each other. Furthermore,
 √ √
 √2 → −√2
στ δ: 3 → −√3
 √
5→− 5
So the Galois group G is isomorphic to Z2 × Z2 × Z2 .

Now, find all fixed fields corresponding to the subgroups of G.


√ √
From
√ above
√ we know that√the√subgroup hσi fixes the field Q( √ √ 3, 5), hτ i fixes √ the √field
Q( 2, 5), also hδi fixes
√ √ Q( 2, 3). Furthermore, hστ i fixes Q( 5, 6), hσδi fixes Q( 3, 10)
and hτ σi fixes Q( 2, 15)....etc. In summary we have

subgroups fixed
√ √fields√
{1} Q( 2,√ 3,√ 5)
{1, σ} Q(√3, √5)
{1, τ } Q(√2, √5)
{1, δ} Q(√2, √3)
{1, στ } Q(√ 5,√ 6)
{1, τ δ} Q(√2, √15)
{1, σδ} √ √ Q(√ 3, 10) √ √
{1, στ δ} Q( 6, 10, 15) ∼
√ = Q( 6, 10)
{1, σ, τ, στ } Q(√5)
{1, σ, δ, σδ} Q(√3)
{1, τ, δ, τ δ} Q(√ 2)
{1, σ, τ δ, στ δ} Q(√15)
{1, τ, σδ, τ σδ} Q( √10)
{1, δ, στ, δστ } Q(√ 6)
{1, στ, τ δ, σδ} Q( 30)
hσ, τ, δi Q
which is the correspondence between
√ the
√ subgroups
√ of the Galois group Z2 × Z2 × Z2 and the
fixed fields. And |Gal(K/Q)| = [Q( 2, 3, 5) : Q] = 8.

14.2.4 Let p be a prime. Determine the elements of the Galois group of xp − 2.

45

By example in 13.4 on page 541, the roots of the √ polynomial xp −√2 are ξ p 2 where ξ is the pth
root of unity. The splitting field for xp − 2 is Q( p 2, ξp ) and [Q( p 2, ξp ) : Q] = p(p − 1).
√ √
p − 2 (all roots ξ p 2 are
Since Q( p 2, ξ) is the splitting field for the
√ separable polynomial
√ x
distinct), so it is Galois over Q. Hence [Q( p 2, ξp ) : Q] =| Gal(Q( p 2, ξp )/Q) |= p(p − 1).

Now, determine the elements of the Galois group. Any automorphism


√ fixes the prime field Q
and is completely determined by its action on the generators p 2 and ξ which must be mapped
to one of the roots of xp − 2 (since xp − 2 is irreducible over Q by Eisenstein at p = 2) by
Proposition 2 in 14.1.
 √
p
√  √ √
2→ξp2 p
2→ p2
Define σ: and τ :
ξ→ξ ξ → ξa
where ξ is the generator of the multiplicative group F∗p with |ξ a | = p − 1. Thus σ p = 1 and
a

τ p−1 = 1.
 √ √ √  √ √ √
a
p
2 → p 2 → ξa p 2 p
2 → ξ p 2 → ξa p 2
Also σ τ : and τ σ:
ξ → ξa → ξa ξ → ξ → ξa
So σ a τ = τ σ.

Hence, the Galois group G = hσ, τ | σ p = τ p−1 = 1, σ a τ = τ σi.

14.2.5 Prove that the Galois group of xp − 2 for p a prime is isomorphic to the
group of matrices  
a b
0 1
where a, b ∈ Fp , a 6= 0.
√ √ √ √
The roots of xp − 2 are p 2, ξ p 2, ξ 2 p 2, · · · ,√ξ p−1 √
p
2 where ξ is a primitive pth root of unity.
p p
Hence the splitting field can be written as Q( 2, ξ 2) (example in 13.4, p541). Any automor-
phism σ ∈ G maps each of these two elements to one the roots of xp − 2 hence can be defined
by  √ √
p
2 → ξb p 2
σa,b :
ξ → ξa

Since every element in G must map these generator √ ξ and p 2 to their√conjugate so there are
at most p(p − 1) choices. And p(p − 1) = |Gal(Q(ξp , p 2)/Q)| = [Q(ξp , p 2) : Q]. So σa,b where
a, b ∈ Zp , a 6= 0 are precisely the elements of G.

Now let A be the group of matrices above. Define a map f : G → A by


 
a b
f : σa,b 7→
0 1

- f is well-defined since if σa,b = σc,d then we have


   
a b c d
=
0 1 0 1

- f is injective: if σ ∈ ker(f ), f (σ) is the identity matrix (note that here the operation of the
matrix group is a multiplication). This implies that a = 1, and b = 0. Hence σ1,0 is an identity
46
in G.
- f is clearly surjective by the way it is defined.
- f is a group homomorphism: for σa,b , σc,d ∈ G, since
 √ √ √
p
2 → ξ b p 2 → ξ bc+d p 2
σc,d · σa,b =
ξ → ξ a → ξ ac
So we have σc,d · σa,b = σbc+d,ac .
    
ac bc + d c d a b
Hence f (σc,d · σa,b ) = f (σac,bc+d ) = = = f (σc,d )f (σa,b )
0 1 0 1 0 1
Therefore we proved that f is an isomorphism.

14.2.10 Determine the Galois group of the splitting field over Q of x8 − 3.



(1) Prove that the splitting field for x8 − 3 is Q( 8 3, ξ), where ξ is a primitive 8th root of unity.

First, find the roots of this polynomial. If α is a root of this equation, then α8 = 3, then
(ξα)8 = 3 where ξ is any primitive 8th root of 1. Hence the solutions of this equation are

ξ a 8th root of 1
8
ξ 3,
Compute ξ explicitly, first note that x8 − 1 = (x4 − 1)(x4 + 1)

= (x√− 1)(x + 1)(x2 + 1)(x4 + 1).
And exercise 13.4.2 proved that the roots for x + 1 are ± 2 ± 22 i. In addition, ±i are the
4 2
√ √ √ √
2 2 2 2
roots of x2 + 1. Hence ξ is precisely (± 2 ± 2 i)i which is again ± 2 ± 2 i.

Let K be the splitting


√ field for x8 − 3 over Q. By definition,
√ K contains
√ all roots√of x8 − 3
8 8
hence contains 3 and ξ which is the ratio of two roots
√ 3 and ξ 3. So K ⊇√Q( 8 3, ξ). On
8

the other hand, all roots are clearly contained in Q( 3, ξ). Therefore, K = Q( 8 3, ξ).
8

√ √ √ √
2

2
Now prove that Q( 8 3, ξ) = Q( 8 3, 2, i) where ξ is a primitive 8th root of 1, i.e. 2 + 2 i.

⊆ is clear. The other containment ⊇ is also true√because
√ 2 = ξ + ξ 7 and i = ξ 2 . Hence we
8
proved that the splitting field field is actually Q( 3, 2, i).
√ √
(2) Prove that [Q( 8 3, 2, i) : Q] = 32.
√ √ √ √ √
Consider the extension of fields Q ⊆ Q( 8 3) ⊆ Q( 8 3, 2) ⊆ Q( 8 3, 2, i)
√ 8
For the first containment, Q( 8 3) has degree
√ √ √x − 3 √
8 over Q since it satisfies which is monic,
irreducible by Eisenstein at p = 3 and has 3 as a root. Also, [Q( 3, √2) : Q( 8 3)] has degree
8 8

at most 2 and
√ is precisely
√ 2 if and only if x2 − 2 is irreducible over Q( 8 3). And this is true if
8
and only if 2 6∈ Q( 3).
√ √ √ √
So we√want to√prove that 2 6∈ Q( 8 3). First, prove that 2 6∈ Q( 4 3). We√know already √
4
that
√ √ 2 ∈
6 Q( 3) (use
√ the same
√ argument in√example(2) √on page √526). So if 2 ∈ Q( 3) =
4 4 4
Q( 3)( 3), then 2 = a + b 3, for a, b ∈ Q( 3). But Q( 3)/Q( 3) is a quadratic extension
hence is a degree
√ 2 Galois
√ extension. So there are two automorphisms: √ an identity and σ,
4
say. Hence σ : 2√7→ − 2. Since √ an automorphism will maps√ a + b 3 to
√ its conjugate, so
4 4 4
we have σ : a + b 3 7→ a − b 3. But we just assumed that 2 = a + b 3, so this implies
47
√ √
σ: 2 7→ a − b 4 3. In addition, automorphism σ fixes Q. Combine all these we have
√ √ √ √ √ √ √
σ(2) = σ( 2 2) = σ( 2)σ( 2) = (a − b 3)(a − b 3) = (a − b 3)2
4 4 4

On the other hand, we have


√ √ √ √ √ √ √
1(2) = 1( 2 2) = 2 2 = (a + b 3)(a + b 3) = (a + b 3)2
4 4 4

√ √
So these√two should be equal. Hence we have (a + b 2)2 = (a − b 2)2 . After simplifying we
get 2ab 4 3 = 0. So either a or b must be 0.
√ √ √
4
If b = 0, a 6= 0, we have2=√ a, a contradiction. If a = 0, b 6
= 0, we get 2 = b 3. Raise
2 √
to both sides, we have b = √
4
, a contradiction since b ∈ Q( 3). Hence we proved that
√ √ 3
2 6∈ Q( 4 3).
√ √
8

Now,
√ proceed√ to the
√ next stage.
√ Want to
√ prove that 2
√ ∈
6 Q( 3). Assuming that 2 ∈
8 4 8 8 4
Q( 3) = Q( 3)( 3), then 2 = a + b 3 for a, √b ∈ Q( √ 3). Use the same method above,
8
we √can √get a contradiction,
√ hence prove that 2 ∈
6 Q( 3). Therefore, we proved that
8 8
[Q( 3, 2) : Q( 3)] = 2.
√ √ √ √ √ √
Finally, Q( 8 3, 2, i) has degree 2 extension over Q( 8 3, 2) since i 6∈ Q( 8 3, 2) and satisfies
the degree 2 irreducible polynomial x2 + 1.
Thus, by the multiplication rule of field extensions, we have

8
√ √
8
√ √
8
√ √
8
√ √
8

8
[Q( 3, 2, i) : Q] = [Q( 3, 2, i) : Q( 3, 2)][Q( 3, 2) : Q( 3)][Q( 3) : Q] = 2·2·8 = 32

(3) Determine the Galois group

Any automorphism
√ √ fixes the prime field Q and is completely determined by its action on the
8
generators 3, 2 and i.
 √ 8

8
 √8

8
 √8

8

 3 → ξ 3 
 3 → 3 
 3 → 3
i√→ i √ i√→ −i√ i√→ i √
  
Define σ: τ: δ:

 2→ 2 
 2→ 2 
 2→− 2

ξ→ξ

ξ→ξ 7 
ξ → ξ5
 √ 8

8
√ √
7 83→ 83

 3 → ξ 3 → ξ
i√→ i → √ −i →√−i √

So we have στ σ:

 2 → 2→ 2→ 2
ξ → ξ → ξ7 → ξ7

which is actually τ . Hence στ σ = τ .

Also, an easy computation gives us σ 8 = τ 2 = 1. So we found G contains a subgroup


H = hσ, τ | σ 8 = τ 2 = 1, στ σ = τ i which is isomorphic to D16 and is a normal subgroup
(with index 2). On the other hand, we see that δ 2 = 1, so G also contains a cyclic subgroup K
of order 2. Note that K is also a normal subgroup of G since σδ = δσ and τ δ = δτ as below:

48
 √ 8
√ √  √ √ √

 3→ 83→ξ83 

8
3→ξ83→ξ83
i√→ i → √i i√→ i →√ −i
 
σδ : √ , δσ : √

 2→− 2→− 2  2→ 2→− 2

ξ → ξ5 → ξ5 ξ → ξ → ξ5
 

and  √ √ √  √ √ √
8

 3→ 83→ 83 

8
3→ 83→ 83
i√→ i → √−i i√→ −i → √ −i
 
τδ : √ , δτ : √

 2→− 2→− 2 
 2→− 2→− 2
ξ → ξ 5 → ξ 35 = ξ 3 ξ → ξ 7 → ξ 35 = ξ 3
 

so δ commutes with hδi and hτ i hence commutes with all elements in G.

Since H and K are normal subgroups of G, so HK is a subgroup of G by Corollary 15 in


section 3.2. But HK has order 32 by Proposition 13 in section 3.2 below

|H||K|
| HK |=
|H ∩ K|

if H and K are finite subgroups of G (note that here H ∩ K = 1). But G has order 32, hence
we must have HK = G.

Now since both H and K are normal subgroup and H ∩ K = 1, by Theorem 9 in section 5,4
we have G = HK ∼= H × K. But H ∼= D16 and K ∼= Z2 , hence we have G ∼
= D16 × Z2 .

14.2.13 Prove that if the Galois group of the splitting field of a cubic over Q is the
cyclic group of order 3 then all the roots of the cubic are reals.

Let K be the splitting field of a cubic polynomial f over Q. Let G denote the Galois group
Gal(K/Q). Suppose G is a cyclic group of order 3. We claim that in this case all roots are
real. Suppose not, since complex roots are conjugate so must be in pair. Let β1 and β2 be the
conjugate complex roots and α the real root. The conjugation induces a transposition that
permutes these two roots. And it is an automorphism of K fixing Q. So the Galois group G
contains an element of order two. But this is a contradiction since by assumption G is cyclic
of order 3 that can’t contain an element of order 2.

Hence, all three roots are real.


p √
14.2.14 Show that Q( 2 + 2) is a cyclic quartic field, i.e. is a Galois extension of
degree 4 with cyclic Galois group.
p √ √ √
Let α = 2 + 2, so α2 = 2 + 2 which is α2 − 2 = 2. Squaring both sides we get
(α2 − 2)2 = 2, or α4 − 4α2 + 4 = 2. So we have α4 − 4α2 + 2 = 0. Therefore, the polynomial
f (x) = x4 − 4x2 + 2 = 0 has α as a root. f (x) is irreducible by Eisenstein at p = 2.

Since f (x) is an irreducible polynomial over Q√ hence separable. Find all roots of f (x). Let
f (x) =px4 − 4x2 + 2 = 0, we have x2 = 2 ± 2. Taking the square root both sides we get

x = ± 2 ± 2 and these are the roots of f (x).

49
p √
Now, show that Q( 2 + 2) is the splitting field of f (x).
p Let√K be the splitting field of f (x).
K must contains all roots hence apparently contain Q( 2 + 2). On the other hand, need to
p √ p √ p √ √2+√2
prove that all roots are in K. Let β = 2 − 2, since β = 2 − 2 = 2 − 2 √ √ =
√ √ 2+ 2
2 √ √ 2 α 2−2 2
p √ . But α2 = 2 + 2 gives 2 = α2 − 2. Hence, β = p √ = = α− ,
2+ 2 p 2+ 2 α α
√ p √
so β is in Q( 2 + 2). Hence all roots ±α and ±β are in K. Thus, K = Q( 2 + 2) is the
splitting field of f (x).

Since K is the splitting field of the separable polynomial f (x), so K/Q is Galois extension. Let
G = Gal(K/Q). Any automorphism in G is completely determined by its action on α. Define
2 σ 2 σ 2 2 α2 − 2 2
σ : α 7→ β(= α − ). So we have α −→ α − −→ (α − ) − 2 = − α2 −2 =
α α α α− α α
√ 2
√ √ α
2 2α 2 − 2α 2 − 2(2 + 2) 2+2 σ 2 2
−√ = √ = √ p √ = −p √ = −α −→ −α − = −α + =
α 2 2α 2( 2 + 2) 2 + 2 −α α
√ √
−α2 + 2 2 2 p √ σ p √
=− =p √ = − 2 − 2 −→ 2 + 2 = α
α α 2+ 2
In summary, we have
σ 2 σ σ σ
α −→ α − −→ −α −→ β −→ α
α
i.e.pσ 4 = 1. Therefore, the Galois group G is isomorphic to cyclic group of order 4. i.e.

Q( 2 + 2) is a cyclic quartic field.
√ √
14.3.6 Suppose K =√Q(θ) = √ Q( D1 , √D2 ) with D1 , D2 ∈ Z, is a biquadratic extension
and that θ = a + b D1 + c D2 + d D1 D2 where a, b, c, d ∈ Z are integers. Prove
that the minimal polynomial mθ (x) for θ over Q is irreducible of degree 4 over Q
but is reducible modulo every prime p. In particular show that the polynomial
x4 − 10x2 + 1 is irreducible in Z[x] but is reducible modulo every prime.

(1) Prove that there are no biquadratic extensions over finite fields.

Suppose K is a biquadratic extension over Fp for prime p, by definition of biquadratic exten-


sion, [K : Fp ] has degree 4 and the Gal(K/Fp ) is isomorphic to Klein 4 group V4 (exercise
14.2.15). But note that for any finite field K of dimension 4 over Fp , then the field K has p4
4
elements and is precisely the splitting field for xp − x by the discussion on 13.5 (page 549-550).
And the finite field, denoted by Fp4 , is unique up to isomorphism. Hence K ∼ = Fp4 . But the
Galois group of finite field extension Fp4 /Fp is a cyclic group of order 4. Since Klein 4 group
is not isomorphic to cyclic group of order 4, a contradiction. Therefore, we proved that there
are no biquadratic extensions over finite fields. 

Prove that the minimal polynomial for θ over Q is irreducible of degree 4 over Q.

Since the biquadratic field extension has degree 4 (exercise 13.2.8), so [Q(θ) : Q] = deg
mθ (x) = 4. And by definition, the minimal polynomial is irreducible. Hence mθ (x) is ir-
reducible of degree 4.

50
However, mθ (x) is reducible modulo every p. Suppose not, suppose it is irreducible mod p.
But since mθ (x) is the minimal polynomial, so we have [Fp (θ) : Fp ] = deg mθ (x) = 4 which is
not possible since we just proved that there are no biquadratic extensions over finite fields.

(2) Show that x4 − 10x2 + 1 is irreducible in Z but not irreducible modulo every prime.

4 2
√ we have proved that the polynomial x − 10x + 1 is
In exercise 13.2.7 in the first homework,

irreducible in Q[x] and has ± 2 ± 3 as roots. By Corollary 6 (the corollary of the Gauss
Lemma) in Section 9.3, then x4 − 10x2 + 1 is irreducible over Z[x].

However, x4 − 10x2 + 1 is reducible modulo every √ prime


√ p. Suppose it is irreducible, then
4 2
x − 10x + 1 is the minimal polynomial for θ = 2 + 3 (it is monic and has θ as a root).
Hence [Fp (θ) : Fp ] = deg mθ (x) = 4 which is a biquadratic extension over Fp . Again this is
not possible since we just proved that there are no biquadratic extensions over finite fields.

14.3.7 Prove that one of 2,3 or 6 is a square in Fp for every prime p. Conclude
that the polynomial
x6 − 11x4 + 36x2 − 36 = (x2 − 2)(x2 − 3)(x2 − 6)
has a root modulo p for every prime p but has no root in Z.

If 2 or 3 is a square in Fp , then we are done since the polynomial has a root mod p. If neither
2 nor 3 is a square in Fp , want to prove that 2 · 3 = 6 is a square in Fp .

By Proposition 18 in section 9.5, the multiplicative group F× p of nonzero elements of Fp is


cyclic. Let F×
p = hσi. Every element in F × is σ k for some integer k. So 2 = σ m and 3 = σ n ,
p
where m, n are integers. Note that σ k is a square if and only if k is even.

But we assume that 2, 3 are not squares, so m, n are both odds. Thus, we have 2 · 3 = σ m σ n =
σ m+n which is a square in Fp since k + m must be even. This implies that 6 is a square in Fp .
i.e. the polynomial has a root mod p for every prime p.
√ √ √
However, this polynomial has no root in Z[x] since all roots ± 2, ± 3 and ± 6 are not in Z.
p √
14.4.1 Determine the Galois closure of the field Q( 1 + 2).
p √ √ √
Let α = 1 + 2, then α2 = 1 + 2. So α2 − 1 = 2. Squaring both sides we have
(α2 − 1)2 = α4 − 2α2 + 1 = 2 hence α4 − 2α2 − 1 = 0. So α satisfies theppolynomial

f (x) = x4 − 2x2 − 1 over Q. To find all roots of f (x), solving for x we get x = ± 1 ± 2.
p √ p √
Now, α is in R, however the other root 1 − 2 is in C. So the field Q( 1 + 2) is clearly
not a splitting field of f (x) hence not a Galois extension of Q.

f (x) is irreducible over Q by Eisenstein at p = 2, hence is separable (Corollary 34 in Sec


p √
13.5). So Q( 1 + 2) is a finite (since α satisfies an irreducible polynomial of degree 4) and
separable extension (by Corollary 39). To find the Galois closure, need to find the splitting
p √ i
field of f (x). Let β = 1 − 2. As mentioned earlier β is in C, more precisely, β = through
α
the following computation.
51
p √ p √

q
1 1 1− 2 1− 2
=p √ p √ = = −i 1 − 2 = −iβ
α 1+ 2 1− 2 i
Hence, all roots of f (x) are ±α and ±β and the splitting field of f (x) is therefore Q(α, i). So
K = Q(α, i) is Galois over Q since it is the splitting field of the separable polynomial f (x) in
Q. pNow, since in a fixed algebraic closure of K any other Galois extension of Q containing

Q( 1 + 2) contains K.

14.4.3 Let F be a field contained in the ring of n × n matrices over Q. Prove that
[F : Q] ≤ n.

Proof

By exercise 13.2.19, the ring of n × n matrices over Q contains fields of degree n over Q. Let
F denote one of such fields. Since F sits inside an n × n matrix ring over Q, F/Q is a finite
extension. By Theorem 25, F/Q is a simple extension, so F = Q(θ) for some θ ∈ F .

Now, since [Q(θ) : Q] = deg mθ (x). And the minimal polynomial must divide the characteris-
tic polynomial which has degree n for an n × n matrix over Q. Hence, the degree of minimal
polynomial can not be greater than n. i.e. [Q(θ) : Q] =degmθ (x) ≤ n. Therefore, we proved
that [F : Q] ≤ n.

14.5.4 Let σa ∈ Gal (Q(ξn )/Q) denote the automorphism of the cyclotomic field of
nth roots of unity which maps ξn to ξna where a is relatively prime to n and ξn is a
primitive nth root of unity. Show that σa (ξ) = ξ a for every nth root of unity.

σ : ξn 7→ ξna is a homomorphism with (a, n) = 1. Let ξ be any nth root of unity, then ξ = ξnm
for some positive integer m. Hence we have
σa (ξ) = σa (ξnm ) = σa (ξn )m = (ξna )m = ξnam = (ξnm )a = ξ a
Therefore, σa (ξ) = ξ a for every nth root of unity.

14.5.10 Prove that Q( 3 2) is not a subfield of any cyclotomic field over Q.

Suppose that Q( 3 2) is a subfield of a cyclotomic field over Q. i.e. there is a field extensions

3
Q ⊆ Q( 2) ⊆ Q(ξn )
for some positive integer n. Since the Galois group of the cyclotomic field extension Q(ξn )/Q
is isomorphic to the multiplicative group (Z/nZ)× which is a cyclic group of order ϕ(n) hence
is abelian. So every subgroup of the Galois group is normal subgroup.

3
Let H be the corresponding
√ subgroup of the field Q( 2), so H is normal
√ in G. By Galois
3
Theory, the Q( √ 2) is Galois over Q. But this is a contradiction since Q( 3 2) is not Galois over

Q because the 3 2 satisfies the irreducible polynomial f (x) = x3 − 2 over √ Q, however, Q( 3
2)
3
is not a splitting field of f (x) since the root ξ3 is not contained in Q( 2).

Therefore, Q( 3 2) is not a subfield of any cyclotomic field over Q.

52
14.5.11 Prove that the primitive nth roots of unity form a basis over Q for the
cyclotomic field of nth roots of unity if and only if n is squarefree.

By the discussion on the bottom of page 598, suppose n = pa11 pa22 · · · pakk is the decomposition
a2 a
p ···p k
of n into distinct prime powers. Since ξn2 k is a primitive pa11 th root of unity. The field
K1 = Q(ξpa1 ) is a subfield of Q(ξn ). Similarly, each of the fields Ki = Q(ξpai ), i = 1, 2, · · · , k
1 i
is a subfield of Q(ξn ). Hence K1 K2 · · · Kk ⊆ Q(ξn ). On the other hand, the composite of the
the fields contains the product ξpa1 ξpa2 · · · ξpak , which is the primitive nth root of unity, hence
1 2 k
contains Q(ξn ). i.e. K1 K2 · · · Kk ⊇ Q(ξn ). Thus, the composite field is Q(ξn ). i.e.
Q(ξn ) = K1 K2 · · · Kk = Q(ξpa1 ) · · · Q(ξpak )
1 k

Since the extension degrees [Ki : Q] = ϕ(pai i ), i = 1, 2, · · · , k and ϕ(n) = ϕ(pa11 )ϕ(pa22 ) · · · ϕ(pakk ),
the degree of the composite of the fields Ki is precisely the product of the degrees of the Ki .
i.e.
[Q(ξ n ) : Q] = [Q(ξpa1 ) · · · Q(ξpak ) : Q] = [Q(ξpa1 ) : Q] · · · [Q(ξpak ) : Q]
1 k 1 k

Now, prove that the primitive nth roots of unity is linearly independent over Q if and only if
n is squarefree.

(⇐)
If n is square free, then n = p. In this case, {1, ξp , ξp2 , · · · , ξpp−2 } is linearly independent since
its minimal polynomial over Q, ξp has degree p−1. Multiplying by ξp we get {ξp , ξp2 , · · · , ξpp−1 },
this is still linearly independent hence they also forms a basis of Q(ξp )/Q.

(⇒)
Suppose n is not square free, i.e. n = pa for a ≥ 2. Then the primitive pa -th roots of
a−1
unity is ξpka where (k, p) = 1. Let ω = ξppa , then ω p = 1 which implies that ω p − 1 =
(ω − 1)(1 + · · · + ω p−1 ) = 0. But ω 6= 1, hence we have
a−1 a−1 a−1 a−1 a−1 (p−1)pa−1
0 = 1 + ξppa + (ξppa )2 + · · · + (ξppa )p−1 = 1 + ξppa + ξp2pa + · · · + ξp a
Multiplying ξpa gives
a−1 +1 a−1 +1 (p−1)pa−1 +1
ξpa + ξppa + ξp2pa + · · · + ξpa
i.e.
a−1 a−1 (p−1)pa−1 +1
ξpa = −(ξppa +1 + ξp2pa +1 + · · · + ξpa )
a
This proves that the primitive p -th roots of unity for a ≥ 2 is linearly dependent. There-
fore, the primitive nth roots of unity {1, ξn , ξn2 , · · · , ξnn−1 } is a basis if and only if n is squarefree.

14.5.12 Let σp denote the Frobenius automorphism x 7→ xp of the finite field Fq of


q = pn elements. Viewing Fq as a vector space V of dimension n over Fp we can
consider σp as a linear transformation of V to V . Determine the characteristic
polynomial of σp and prove that the linear transformation σp is diagonalizable over
Fp if and only if n divides p − 1, and is diagonalizable over the algebraic closure of
Fp if and only if (n, p) = 1.
n
For each x ∈ Fp , we have xp − x = 0. So the transformation σp satisfies the polynomial
X n − 1 = 0. Since this polynomial is of degree n, it must be the characteristic polynomial.
The linear transformation σp is diagonalizable over Fp if and only if X n − 1 splits into linear
53
factors (not necessarily distinct) in Fp , and this is true if and only if Fp contains all nth roots
of unity. And this is true if and only if F× p contains a cyclic group of order n, and is true if
×
and only if n | |Fp | = p − 1.

The linearly transformation σp is diagonalizable over Fp = n≥1 Fpn if and only if X n − 1


S

is separable (guarantee all roots are distinct?). And X n − 1 is separable if and only if it is
relatively prime to nX n−1 , which is true if and only if nX n−1 is nonzero in Fp , i.e. p - n, or
(p, n) = 1.

14.6.17 Find the Galois group of x4 − 7 over Q explicitly as a permutation group


on the roots.
√ √ √ √ √ √
Since x4 − 7 = (x + i 4 7)(x − i 4 7)(x + 4 7)(x − 4 7). So ± 4 7, ±i

4
7 are the four roots of the
4
polynomial x − 7 over Q. Prove that the splitting field is Q(i, 7).4

Let K√be the √ splitting field of f (x) over Q. √K contains all roots hence contain the ratio of two
4 4
roots 7,√ i 7 which is i. Hence
√ K ⊇ Q(i, 4 7). On√ the other hand, all roots above are in this
field Q(i, 4 7). So K ⊆ Q(i, 4 7), hence K = Q(i, 4 7).

Consider the extension of fields


√4

4
Q ⊆ Q( 7) ⊆ Q(i, 7)
√ √
The field extension Q( 4 7)/Q has degree at most 4 since 4 7 satisfies the polynomial√x4 − 7.
But x4 − 7 is irreducible by Eisenstein
√ at p = 7 hence is the minimal polynomial of 4 7 over
4
Q. Therefore,
√ √ the extension Q( 7)/Q has degree precisely 4. Furthermore,√the extension
Q(i, 4 7)/Q( 4 7) clearly has degree 2 since i is not contained in the field Q( 4 7). Hence by
multiplication rule of fields extension we have

4

4
√4
√4
[Q(i, 7) : Q] = [Q(i, 7) : Q( 7)] · [Q( 7) : Q] = 2 · 4 = 8

Hence, the Galois group G = Gal(Q(i, 4 7)/Q) has degree 8.

Any automorphism in G is completely determined by its action on the generators 4 7 and i.
First note that the 4th root of unity is i. So we define
 √4
√  √ √
7→i47 4
7→ 47
σ: and τ :
i→i i → −i
√ √ √ √ √
σ : 4 7 7→ i 4 7 7→ i2 4 7 7→ i3 4 7 7→ 4 7 and σ fixes i hence σ 4 = 1. On the other hand, τ
Since √
fixes 4 7 and τ : i 7→ −i 7→ i so τ 2 = 1.
 √ 4
√ √
7→ 47→i47
Also, στ :
i → −i → −i
 √ √ √ √ √ √
3
4
7 → i 4 7 → i 4 7 → i2 4 7 → i3 4 7 → i 4 7
And τ σ :
i → i → i → i → −i
So στ = τ σ 3 . Hence the Galois group G is isomorphic to a Dihedral group of order 8
D8 = {σ, τ | σ 4 = τ 2 = 1, στ = τ σ 3 }.

To describe the elements of the Galois


√ group explicitly
√ as a permutation
√ group
√ on the roots,
4 4 4 4 4
denote the roots of x − 7 by α1 = 7, α2 = i 7, α3 = − 7 and α4 = −i 7. Or simply
54
√ √ √ √
denote 1 = 4 7, 2 = i 4 7, 3 = − 4 7 and 4 = −i 4 7. So as a permutation group, σ = (1234),
τ = (24). Further computation gives

σ 2 = (1234)(1234) = (13)(24)
σ 3 = (1234)(13)(24) = (1432)
τ σ = (24)(1234) = (14)(23)
τ σ 2 = (24)(13)(24) = (13)
τ σ 3 = (24)(1432) = (12)(34)

In summary, we have G = {1, (13), (24), (13)(24), (12)(34), (14)(23), (1234), (1432)} which is
isomorphic to D8 .

15.1.2 Show that each of the following rings are not Noetherian by exhibiting an
explicit infinite increasing chain of ideals:
(a) the ring of continuous real valued functions on [0, 1].
(b) the ring of all functions from any infinite set X to Z/2Z.

(a) Let R be the ring of continuous real valued functions on [0, 1]. And let Jm be the collection
1 1
of closed sub-intervals on [0, 1] such that Ji = [ 21 − 2i 1
, 2 + 2i ], then J1 ⊃ J2 ⊃ · · · Jn ⊃ · · · .
Now, let Ii be subsets of R as follows:

I1 = {f ∈ R | f = 0 on J1 }
I2 = {f ∈ R | f = 0 on J2 }
..
.
In = {f ∈ R | f = 0 on Jn }
..
.
Prove that all Ii , i = 1, 2, · · · are ideals of R.

- Ii is nonempty since zero map is a continuous function.


- Ii is closed under multiplication: for f, g ∈ Ii , both f = 0 and g = 0 on Ji . So we have
f g = 0 on Ji , hence f g ∈ Ii .
- Ii is closed under subtraction: for f, g ∈ Ii , clearly f − g = 0. So f − g ∈ Ii .
Hence I1 is a subring.

Furthermore, for any function h ∈ R and f ∈ Ii , we have hf = hf = 0 on Ji . So hf and f h


are in Ii . Therefore, we proved that Ii is an ideal for all i.

Now, the collection of {Ji } is an infinite descending chain of sub-intervals in [0, 1]. This gives
us a collection of ideals vanishing on {Ji } which is an infinite ascending chain. i.e.
I1 ⊂ I2 ⊂ I3 ⊂ · · ·
is an infinite ascending chain of ideals.

Hence R is not Noetherian.

(b) Let R be the ring of all functions f : X → Z/2Z. And let Ji be the collection of points in
X such that Ji = X − {xα1 , xα2 , · · · , xαi } where xαj ∈ X and αj ∈ I an index set. Then we
55
obtain an descending chain of subsets J1 ⊃ J2 ⊃ · · · Jn ⊃ · · · in X.

Now, let Ii be subsets of R as follows:

I1 = {f ∈ R | f = 0 on J1 }
I2 = {f ∈ R | f = 0 on J2 }
..
.
In = {f ∈ R | f = 0 on Jn }
..
.
Same argument as in (a) will prove that all Ii , i = 1, 2, · · · are ideals of R.

- Ii is nonempty since zero map is a continuous function.


- Ii is closed under multiplication: for f, g ∈ Ii , both f = 0 and g = 0 on Ji . So we have
f g = 0 on Ji , hence f g ∈ Ii .
- Ii is closed under subtraction: for f, g ∈ Ii , clearly f − g = 0. So f − g ∈ Ii .
Hence I1 is a subring.

Furthermore, for any function h ∈ R and f ∈ Ii , we have hf = hf = 0 on Ji . So hf and f h


are in Ii . Therefore, we proved that Ii is an ideal for all i.

Now, the collection of {Ji } is an infinite descending chain in X. This gives us a collection of
ideals vanishing on {Ji } which is an infinite ascending chain. i.e.
I1 ⊂ I2 ⊂ I3 ⊂ · · ·
is an infinite ascending chain of ideals.

Hence R is not Noetherian.

15.1.5 Suppose M is a Noetherian R-module and ϕ : M → M is an R-module endo-


morphism of M . Prove that ker(ϕn )∩ image(ϕn ) = 0 for n sufficiently large. show
that if ϕ is surjective, then ϕ is an isomorphism.

(1) Prove that ker(ϕn )∩ image(ϕn ) = 0 for n sufficiently large.

⊇ is clear.
⊆ For an element a ∈ ker ϕn ∩imageϕn , there exists an element b ∈ M such that ϕn (b) = a.
Also, ϕn (a) = 0. This implies that ϕn (ϕn (b)) = ϕ2n (b) = 0. So we get b ∈ ker ϕ2n = ker ϕn
since M is a Noetherian R-module so that the ascending chain of ideals ker ϕ stabilize for
sufficiently large n.

Therefore, ϕn (b) = 0 = a hence a = 0. This proves the containment ⊆. Hence, ker(ϕn )∩


image(ϕn ) = 0 for n sufficiently large.

(2) Prove that if ϕ is surjective, then ϕ is an isomorphism.

If ϕ is surjective, ϕn is surjective so imageϕn = M . But we just proved that ker(ϕn )∩


image(ϕn ) = 0 for n sufficiently large. This means that ker(ϕn ) ∩ M = 0 for n sufficiently
56
large. Hence ker ϕn = 0. Note that ker(ϕ) ⊆ ker(ϕ2 ) ⊆ · · · , hence ker ϕ = 0. Therefore, ϕ is
injective and ϕ is an isomorphism.

15.1.7 Prove that submodules, quotient modules, and finite direct sums of Noe-
therian R-modules are again Noetherian R-modules.

Let M be a Noetherian R-module and N be a submodule of M . If N 0 is a submodule of N ,


since N 0 is also a submodule of M , hence is finitely generated. So N is Noetherian. Every
submodule of M/N has the form L/N where L is a submodule of M with N ⊂ L ⊂ M . Since
M is Noetherian, L is finitely generated, and the reduction of those generators mod N will
generate L/N as an R-module. Finally, 0 → M1 → M1 ⊕ M2 → M2 → 0 is an exact sequence,
if M1 and M2 are Noetherian, by 15.1.6 M1 ⊕M2 is Noetherian. Then by induction we are done.

15.1.8 If R is a Noetherian ring, prove that M is a Noetherian R-module if and


only if M is a finitely generated R-module.

Suppose {x1 , · · · , xn } is generators for M . Define a ring homomorphism ϕ : Rn → M by


(a1 , · · · , an ) 7→ a1 x1 + · · · + an xn . Then ϕ is surjective and we have an exact sequence

0 → ker ϕ → Rn → M → 0.

Since R is Noetherian, Rn is Noetherian by 15.1.7 (direct sum of Noetherian modules is Noe-


therian). Then by 15.1.6 M is Noetherian.

The other direction is clear since from the definition of a Noetherian module, an R-module
that is Noetherian has to be finitely generated.

15.1.11 Suppose R is a commutative ring in which all the prime ideals are finitely
generated. Proves that R is Noetherian.
(a) Prove that if the collection of ideals of R that are not finitely generated is
nonempty, then it contains a maximal element I, and that R/I is a Noetherian
ring.
(b) Prove that there are finitely generated ideals J1 and J2 containing I with
J1 J2 ⊆ I and that J1 J2 is finitely generated.
(c) Prove that I/J1 J2 is a finitely generated R/I−submodule of J1 /J1 J2 .
(d) Show that (c) implies the contradiction that I would be finitely generated over
R and deduce that R is Noetherian.

(a) The proof is basically similar to the proof of Proposition 11 in 7.4 which shows that in a
ring with identity every ideal is contained in a maximal idea. Let S 6= φ be the collection of
S by inclusion. If C is
ideals of R that are not finitely generated. Then S is partially ordered
a chain in S, defined I to be the union of all ideals in C. i.e. I = A∈C A. The same proof
as Proposition 7.4 shows that I is an ideal and actually I is a proper ideal. Hence each chain
has an upper bound in S. By Zorn’s Lemma, S has a maximal element I. So I is not finitely
generated.

This implies that all ideals in the quotient R/I are finitely generated. Hence by Theorem 2,
R/I is Noetherian.

57
(b) By assumption all prime ideals of R are finitely generated, so the maximal element I in S
found in (a) is not finitely generated hence is not prime. So there are two elements a, b of R\I
such that ab is contained in I. Let J1 = I + (a) and J2 = I + (b), then J1 , J2 contains I. Since
both J1 and J2 are not contained in I, so they are not in S hence are finitely generated. So
their product J1 J2 is also finitely generated.

(c) Since (b) proved that J1 J2 ⊆ I ⊆ J1 ⊆ R, then by Lattice Isomorphism Theorem we have

1 ⊆ I/J1 J2 ⊆ J1 /J1 J2 ⊆ R/J1 J2

so I/J1 J2 is a R−submodule of J1 /J1 J2 and can be made into a R/I-submodule by defining


the action of R/I on I/J1 J2 by (r + I)m = rm for each m ∈ I/J1 J2 and r ∈ R. Hence I/J1 J2
is a R/I-submodule of J1 /J1 J2 .

Now, since in (a) we proved that R/I is Noetherian, then R/I is a Noetherian module over
itself. Hence its quotient (R/I)/(J1 J2 /I) ∼
= R/J1 J2 is Noetherian by exercise 15.1.7. Then
from above, I/J1 J2 is a submodule of R/J1 J2 , hence again Noetherian. Thus, I/J1 J2 is a
Noetherian R/I-submodule of J1 /J1 J2 . By exercise 15.1.8, I/J1 J2 is finitely generated.

(d) In (c) we prove that I/J1 J2 is a finitely generated R/I-submodule of J1 /J1 J2 . Since J1 J2
is finitely generated by (b). But this implies that I is a finitely generated R-submodule by
exercise 10.3.7. i.e. I is a finitely generated ideal of R. This is a contradiction since I is an
ideal in the set S consisting of all ideals that are not finitely generated. Hence we conclude
that R is Noetherian.

15.1.26 Let V = Z(xz − y 2 , yz − x3 , z 2 − x2 y) ⊆ A3 .


(a) Prove that the map ϕ : A1 → V defined by ϕ(t) = (t3 , t4 , t5 ) is surjective mor-
phism.
(b) Describe the corresponding k-algebra homomorphism ϕ̃ : k[V ] → k[A1 ] explic-
itly.
(c) Prove that ϕ is not an isomorphism.

(a) ϕ is a morphism since there are polynomials f1 = t3 , f2 = t4 and f3 = t5 ∈ k[A1 ] such that
ϕ(t) = (f1 (t), f2 (t), f3 (t)) = (t3 , t4 , t5 ) for all t ∈ A1 .

To prove that ϕ is surjective, assume that (x, y, z) 6= (0, 0, 0) (if one of x, y, z is zero, the other
two must be zero. hence V = A3 ). So let t = xy . Then we have

y  y   y3 y4 y5 
3 y 4 y 5
ϕ( ) = ( ) , ( ) , ( ) = , ,
x x x x x3 x4 x5

Since by definition V = Z(xz − y 2 , yz − x3 , z 2 − x2 y) = {(x, y, z) ∈ A3 | fi = 0}, where


f1 = xz − y 2 , f2 = yz − x3 and f3 = z 2 − x2 y. Solving fi = 0 for i = 1, 2, 3 we obtain xz = y 2 ,
yz − x3 and z 2 = x2 y. These give us the followings

y3 (xz)y zy x3
= = = =x
x3 x3 x2 x2

y4 x2 z 2 xz y2
= = = =y
x4 xyz y y
58
y5 x2 z 2 y
5
= =z
x yzx2
Hence we have
y
ϕ(t) = ϕ( ) = (x, y, z)
x
so that ϕ is surjective.

(b) The morphism ϕ : A1 → V induces an associated k-algebra homomorphism ϕ̃ : k[V ] →


k[A1 ] defined by ϕ̃(f ) = f ◦ ϕ. And by definition, k[V ] = k[A3 ]/(I) = k[x, y, z]/(xz − y 2 , yz −
x3 , z 2 − x2 y). So explicitly, ϕ̃ maps f to f ◦ ϕ(t) = f (t3 , t4 , t5 ). i.e. ϕ̃ : f (x, y, z) 7→ f (t3 , t4 , t5 ).
Or

 x 7→ x3
ϕ̃: y 7→ x4
z 7→ x5

(c) Note that the image of ϕ̃ is the subalgebra k[x3 , x4 , x5 ] = k + x3 k[x] of k[x]. In particular,
ϕ̃ is not surjective (for example, x2 is not in k + x3 k[x]), hence ϕ̃ is not an isomorphism. Then
by Theorem 6(4), ϕ is not an isomorphism.

15.2.3 Prove that the intersection of two radical ideals is again a radical ideal.

Lemma Let I and J be ideals in the ring R. Then rad (I ∩ J) =rad I∩ rad J.

Proof of Lemma: rad (I ∩ J) = {a ∈ R | ak ∈ I ∩ J for some k ≥ 1} = {a ∈ R | ak ∈


I for some k ≥ 1} ∩ {a ∈ R | ak ∈ I for some k ≥ 1} = rad I ∩ rad J. .

So now, let I, I be two radical ideals. So rad I = I and rad J = J. Prove that I ∩ J is a radical
ideal. i.e. prove that rad(I ∩ J) = I ∩ J. Indeed, rad (I ∩ J)=rad I∩ rad J (by Lemma) =I ∩ J.

Hence rad (I ∩ J) = I ∩ J, the intersection of two radicals is a radical ideal.

15.2.4 Let I = m1 m2 be the product of the ideals m1 = (x, y) and m2 = (x − 1, y − 1)


in F2 [x, y]. Prove that I is a radical ideal. Prove that (x3 − y 2 ) is a radical ideal in
F2 [x, y].

First prove that I is a radical ideal, i.e. prove that rad I = I. It’s obviously that I ⊆ rad I.
It remains to show that

Since F2 [x, y]/m1 ∼


= F2 so m1 is a maximal ideal. Similarly F2 [x, y]/m2 ∼
= F2 . So both m1 and
m2 are maximal ideals hence are prime, so they are radical by Corollary 13.

Now, rad (I) = rad (m1 m2 ) = rad (m1 ) ∩ rad (m2 ) = m1 ∩ m2 since m1 , m2 are radical.

But m1 and m2 are comaximal in R since for any r ∈ R, r is a polynomial of variables x, y


with coefficient either 0 or 1. But each term xi y j for i > 0, j > 0 can be written as
xi y j = (x − 1)xi−1 y j + (y − 1)xi−1 y j−1 + xi−1 y j−1
hence xi y j ∈ (x, y) + (x − 1, y − 1).

59
If i = j = 0, i.e. the polynomial is a constant 1, then 1 = (x − 1) + x + 2 ∈ (x, y) + (x − 1, y − 1).
On the other hand, x ∈ (x, y) + (x − 1, y − 1) and y ∈ (x, y) + (x − 1, y − 1). Hence m1 = (x, y)
and m2 = (x−1, y −1) are comaximal i.e. m1 +m2 = R so m1 m2 = m1 ∩m2 by exercise 7.3.34.

Return to what we just proved earlier that rad I = m1 ∩ m2 . Now since m1 ∩ m2 = m1 m2 .


Hence we have rad I = m1 m2 = I. Namely, I is a radical ideal.

Finally, the ideal (x3 − y 2 ) is prime in k[x, y] for any field k hence prime in F2 [x, y]. Therefore,
(x3 − y 2 ) is a radical ideal in F2 [x, y].

15.2.9 Prove that for any field k the map Z in the Nullstellansatz is always sur-
jective and the map I in the Nullstellansatz is always injective. Give examples
(over a field k that is not algebraically closed) where Z is not injective and I is
not sujective.

(1) Prove that the map Z is always surjective and the map I is always injective.

By Properties of Z in 15.1, if V = Z(I) is an algebraic set, then V = Z(I(V )). i.e. Z · I is an


identity map which implies that I is injective and Z is surjective.

(2) Example that I is not surjective.

Let I = (x2 + 1) be an ideal of R[x]. I is a maximal ideal since R[x]/I = C. So I is prime


hence is a radical ideal. But I has no zeros in R, so the map I is not surjective.

(3) Example that Z is not injective

Let I1 = (1) and I2 = (x2 + 1) be ideals of R[x]. Then I1 6= I2 . However, Z(I1 ) = Z(I2 ) = φ.
Hence Z is not injective.

15.2.28 Prove that each of the following rings have infinitely many minimal prime
ideals, and that (0) is not the intersection of any finite number of these.
(a) the infinite direct product ring Z/2Z × Z/2Z × · · · .
(b) k[x1 , x2 , · · · ]/(x1 x2 , x3 x4 , · · · , x2i−1 x2i , · · · ), where x1 , x2 , · · · are independent vari-
ables over the field k.

(a) Consider the ideals I1 = h(0, 1, 1, · · · )i, I2 = h(1, 0, 1, · · · )i, Ii = h(1, · · · , 1, 0, 1, · · · )i. i.e. Ii
is an ideal generated by an element with all 1s on all coordinates except 0 in the ith coordinate,
in the ring R = Z/2Z × Z/2Z × · · · . These ideals are prime since R/Ii = Z/2Z is an integral
domain. Also, these ideals are minimal. If there is a prime ideal J with J ⊂ Ii and J 6= Ii ,
then J must have two or more of zeros, for example, if J has two zeros in the coordinates. i.e.
J = (1, · · · , 1, 0, 1, · · · , 1, 0, 1, · · · ) then R/J ∼
= Z/2Z × Z/2Z which is not an integral domain
since there is a zero divisor when in the product (i.e. a = (1, 0) and b = (0, 1) are nonzero in
Z/2Z × Z/2Z, but ab = 0). Similar for the case that more than two zeros in the coordinate
of ideal J. So J can not be prime, a contradiction. Therefore, Ii for i = 1, 2, · · · are minimal
prime ideals in R.

Now prove that (0) is not the intersection of any finite number of these ideals. i.e. (0) 6= N
T
i=1 Ii
for some positive integer N . Indeed, the element (0, · · · , 0, 1, 1, 1, · · · , ) with first N position
60
all 0s is contained in the intersection. Hence (0) is not the intersection of any finite number of
these ideals.

(b) First, consider the following ideals


(x1 , x2 , x3 , x7 , · · · ) generated by xi where i is odd.
(x2 , x4 , x6 , x8 , · · · ) generated by xi where i is even.
(x1 , x4 , x5 , x8 , · · · ) the generator have indexes rotating between odd and even.
(x2 , x3 , x6 , x7 , · · · ) same as above.
..
.

In general, let Ik = (xk12 , xk34 , · · · ) where xk12 is a choice from {x1 , x2 } and xk34 is chosen
from {x3 , x4 } and so on. So there are infinitely many combinations of choices hence there are
infinitely many such ideals in R = k[x1 , x2 , · · · ].

Ik are all prime since if we let J = (x1 x2 , x3 x4 , · · · , x2i−1 x2i , · · · ), by the Third Isomorphism
Theorem we have
R/J ∼ R
=
Ik /J Ik
which is an integral domain (for example, k[x1 , x2 , · · · ]/(x1 , x3 , x5 , · · · ) ∼
= k[x2 , x4 , x6 , · · · ] is an
integral domain). Hence Ik /J is a prime ideal in R/J.

Now, claim that Ik /J are minimal. Since Ik is an ideal such that each coordinate was chosen
form the pair x2i−1 , x2i , so suppose there is an ideal I in R such that I/J strictly contained
in Ik /J, i.e. I/J ⊂ Ik /J, then I/J must have zero in at least one of the coordinates, say x2i−1
x2i = 0. But note that the quotient I/J just means that J is contained in I. And we know
that J = (x1 x2 , x3 x4 , · · · , x2i−1 x2i , · · · ). Hence this is a contradiction since J can not sit inside
I. So we proved that Ik /J is the minimal ideal.

Finally, clearly (0) is not the intersection of any finitely many of these ideals Ik /J. Suppose
(0) = N
T
I /J, but we know the product of finitely many ideals is contained in their product.
QNk=1 k TN QN
i.e. k=1 k /J ⊆
I k=1 Ik /J. Hence we have k=1 Ik /J = 0 which is not possible since no
coordinate in Ik /J is zero for all k.

15.2.33 Let I = (x2 , xy, xz, yz) in k[x, y, z]. Prove that a primary decomposition of
I is I = (x, y) ∩ (x, z) ∩ (x, y, z)2 , determine the isolated and embedded primes of I,
and find rad I.

(1) Prove that (x, y), (y, z) and (x, y, z)2 are primary.

Since k[x, y, z]/(x, y) ∼


= k[z] which is an integral domain, so (x, y) is prime hence is primary.
Similarly, k[x, y, z]/(y, z) ∼
= k[x], so (y, z) is primary. The ideal (x, y, z)2 is primary since it is
a power of maximal ideal (x, y, z) (since k[x, y, z]/(x, y, z) ∼
= k is a field).

(2) Find isolated and embedded prime ideals.

As above k[x, y, z]/(x, y) ∼


= k[z] is an integral domain, so (x, y) is prime. But prime ideals are
radical, so rad(x, y) = (x, y). Similarly, rad(y, z) = (y, z). i.e. (x, y) and (y, z) are themselves
the associated prime ideals. Since (x, y) and (y, z) do not contain any other prime ideals of I
61
so they are isolated prime ideals. The other associated prime ideal is rad(x, y, z)2 = (x, y, z).
But (x, y, z) is clearly not an isolated prime since it does contain other ideal (e.g. (x, y)), hence
is the embedded prime ideal.

(3) Prove that I = (x2 , xy, xz, yz) = (x, y) ∩ (y, z) ∩ (x, y, z)2 .
X
The ideal I1 = (x, y) = f x+gy for f, g ∈ k[x, y, z] and it consists of elements {x, y, xy, yz, xz
x , y , xyz, · · · }. Similarly, the ideal I2 = (x, z) consists of elements {x, z, xz, xy, yz, x2 , z 2 , xyz, · · · }
2 2

and I3 = (x, y, z)2 consists of elements {x2 , y 2 , z 2 , xy, yz, xz, xyz, · · · }.

To see the intersection I1 ∩I2 ∩I3 , note that x is contained in I1 , I2 but not I3 , hence is not con-
tained in the intersection. Similarly, elements y, z are not contained in the intersection. Now
look at the possible degree-two elements, we see that xy, yz, xz are clearly in the intersection.
In addition, x2 is also in the intersection, but not y 2 nor z 2 . All degree 3 or higher monomial
can be generated by these elements. Hence we conclude that the intersection of these three
ideals is {x2 , xy, yz, xz}. i.e. I = (x2 , xy, xz, yz) = (x, y) ∩ (y, z) ∩ (x, y, z)2

(4) Find the rad I.

The radical ideal of I is the intersection of the isolated primes of I by Corollary 22. Hence
radI = (x, y) ∩ (x, z) which is (x, yz). The reason (x, y) ∩ (x, z) = (x, yz) is similar to (3).
We had (x, y) = {x, y, xy, yz, xz x2 , y 2 , xyz, · · · } and (x, z) = {x, z, xz, xy, yz, x2 , z 2 , xyz, · · · },
hence (x, y) ∩ (x, z) = (x, yz) since all degree 3 or higher elements can be generated by these
two generators.

15.2.39 Fix an element a in the ring R. For any ideal I in the ring R let Ia = {r ∈
R | ar ∈ I}.
(a) Prove that Ia is an ideal and Ia = R if and only if a ∈ I.
(b) Prove that (I ∩ J)a = Ia ∩ Ja for ideals I and J.
(c) Suppose that Q is a P -primary ideal and that a 6∈ Q. Prove that Qa is a P -
primary ideal and that Qa = Q if a 6∈ P .

(a) First, prove that Ia is an ideal.


- Ia is nonempty: since aI ⊆ I, so I ⊆ Ia , hence Ia is nonempty.
- Ia is closed under multiplication: for r, s ∈ Ia , then r, s ∈ R and ar, as ∈ I. So a(rs) =
(ar)s ∈ I, hence rs ∈ Ia . Thus, Ia is closed under multiplication.
- Ia is closed under subtraction: for r, s ∈ Ia , then a(r − s) = ar − as ∈ I hence r − s ∈ Ia .
Hence Ia is a subring.

Now for any element c ∈ R, and r ∈ Ia , we have (ac)r = a(cr) ∈ I since cr ∈ I. So cIa ⊆ Ia
for any c ∈ R. Similarly we can prove that Ia c ⊆ Ia . Hence Ia is an ideal.

Prove that Ia = R if and only if a ∈ I.


This is clear since if Ia = R, then aR ⊆ I which implies that a = a · 1 ∈ I. On the other hand,
a ∈ I gives aR ⊆ I by definition of ideal. Hence Ia = R.

(b) Since ar ∈ I ∩ J if and only if ar ∈ I and ar ∈ J. Therefore, we have


(I ∩ J)a = {r ∈ R | ar ∈ I ∩ J} = {r ∈ R | ar ∈ I and ar ∈ J}

62
= {r ∈ R | ar ∈ I} ∩ {r ∈ R | ar ∈ J} = Ia ∩ Ja

(c) If Q is P -primary, then radQ = P . First, prove that Qa is primary. Since Qa = {r ∈ R |


ar ∈ Q}. If xy ∈ Qa and x 6∈ Qa , want to prove that y n ∈ Qa for some positive integer n.
Since xy ∈ Qa , then a(xy) = (ax)y ∈ Q. But Q is P -primary, and x 6∈ Qa which implies that
ax 6∈ Q so y n ∈ Q for some positive integer n. So we have ay n ∈ Q. Therefore Qa is primary.

Now prove that Qa is P -primary. i.e. radQa = P .

Since radQa = {r ∈ R | rk ∈ Qa for some k ≥ 1} = {r ∈ R | ark ∈ Q for some k ≥ 1}. But


since a 6∈ P and Q is contained in radQ = P , hence a 6∈ Q. This implies that (rk )l ∈ Q for
some positive integer l (because ark ∈ Q). Thus, radQa ⊆ rad Q. On the other hand, if r ∈
radQ, then rk ∈ Q for some positive integer k, so ark ∈ Q, hence rk ∈ Qa . Hence, radQ ⊆
radQa . Therefore, we have radQa = radQ = P .

Finally, prove that Qa = Q if a 6∈ P . One containment is clear, Q ⊆ Qa . For the other


containment, if r ∈ Qa , then ar ∈ Q. By assumption a 6∈ Q, we claim that r ∈ Q. Suppose
on the contrary r 6∈ Q, but ar ∈ Q and Q is primary, so ak ∈ Q ⊆ P (since Q is primary,
P =radQ is the smallest prime ideal containing Q). Now, ak = ak−1 a ∈ P , but a 6∈ P , so
ak−1 must be in P . Then breaking down to next stage ak−2 a ∈ P obtaining that ak−1 ∈ P .
Eventually we will end up getting a ∈ P which is a contradiction since a is not in P . Hence
we prove that r ∈ Q. Therefore, Qa = Q if a 6∈ P .

15.2.40 With notation as in the previous exercise, suppose I = Q1 ∩ · · · ∩ Qm is min-


imal primary decomposition of the ideal I and let Pi be the prime ideal associated
to Qi .
(a) Prove that Ia = (Q1 )a ∩ · · · ∩ (Qm )a and that rad(Ia ) =rad((Q1 )a ) ∩ · · · ∩ rad((Qm )a ).
(b) Prove that rad(Ia ) is the intersection of the prime ideals Pi for which a 6∈ Qi .
(c) Prove that if rad(Ia ) is a prime ideal then rad(Ia ) = Pj for some j.
(d) For each i = 1, · · · , m, prove that rad(Ia ) = Pi for some a ∈ R.
(e) Show from (c) and (d) that the associated primes for a minimal primary de-
composition are precisely the collection of prime ideals among the ideals rad(Ia )
for a ∈ R, and conclude that they are uniquely determined by I independent of
the minimal primary decomposition.

(a) Since I = Q1 ∩ · · · ∩ Qm , clearly Ia = (Q1 ∩ · · · ∩ Qm )a . Part (b) in previous exercise


proved that (Q1 ∩ Q2 )a = (Q1 )a ∩ (Q2 )a by induction on m we have (Q1 ∩ · · · ∩ Qm )a =
(Q1 )a ∩ · · · ∩ (Qm )a . Hence Ia = (Q1 )a ∩ · · · ∩ (Qm )a .

Now, since Ia = (Q1 )a ∩ · · · ∩ (Qm )a , we have

rad(Ia ) = rad((Q1 )a ∩ · · · ∩ (Qm )a ) = rad((Q1 )a ) ∩ · · · ∩ rad((Qm )a )

by exercise 15.2.2(c).

(b) For a 6∈ Qi , by previous exercise, (Qi )a is P -primary, so rad(Qi )a = Pi is prime. As-


sume that among these Qi , i = 1, 2, · · · , m there are k of them (k ≤ m) such that a is
not contained in these Qi . Renumber these indices to be 1, 2, · · · , k. Then by (a) we have
rad(Ia ) =rad((Q1 )a ) ∩ · · · ∩ rad((Qk )a ) i.e. rad(Ia ) = P1 ∩ · · · ∩ Pk , the intersection of prime
63
ideals Pi for which a 6∈ Qi .

(c) If rad(Ia ) is a prime ideal, it is irreducible. Therefore, rad(Ia ) = Pj for some j.

(d) In a minimal primary decomposition, no primary ideal contains the intersection of the
remaining primary ideals, so forTeach i we can pick an element a in the intersection of Qj for
all j 6= i but a 6∈ Qi . i.e. a ∈ j6=i Qj but a 6∈ Qi . Then by (b), for this element a we have
rad(Ia ) = Pi .

(e) From all above, we proved that the associated primes Pi for a minimal primary decom-
position are precisely the collection of prime ideals among the ideals rad(Ia ) for a ∈ R, i.e.
{rad(Ia ) | a ∈ R} = {Pi }. Hence these prime ideals are uniquely determined by I independent
of the minimal primary decomposition.

16.1.4 Prove that an Artinian integral domain is a field.

Let R be an Artinian integral domain, and a 6= 0 an element in R. We have a descending chain


of ideals (a) ⊇ (a2 ) ⊇ · · · . Since R is Artinian, the descending chain stabilizes. i.e. there is
n ∈ N such that (an ) = (an+1 ). Hence, there is an element r ∈ R such that an = an+1 r. This
means, an = an · ar which implies that 1 = ar since a 6= 0 and R is an integral domain so R
has the cancellation property.

Therefore an Artinian integral domain is a field.

16.1.8 Let M be a maximal ideal of the ring R and suppose that M n = 0 for some
n ≥ 1. Prove that R is Noetherian if and only if R is Artinian.

Consider the filtration R ⊇ M ⊇ · · · ⊇ M n−1 ⊇ M n = 0, observe that each successive quotient


M i /M i+1 , i = 0, · · · , n − 1 is a module over the field R/M . i.e. each of these quotients is a
finite dimensional vector space over the field F . This implies that M i /M i+1 is Noetherian as
well as Artinian by exercise 16.1.7.

15.1.6 and 16.1.6 say that if 0 → M 0 → M → M 00 → 0 is an exact sequence of R-modules.


Then M is a Noetherian (Artinian) R-module if and only if M 0 and M 00 are Noetherian (Ar-
tinian) R-modules.

we have
0 −→ M i+1 −→ M i −→ M i /M i+1 −→ 0

0 −→ M −→ R −→ R/M −→ 0

Now, if R is Noetherian, its submodule M and quotient module are also Noetherian by exer-
cise 15.1.7. Hence by this exercise again the submodule M 2 of M and the quotient M 2 /M are
Noetherian. Inductively, we have all M i and M i /M i+1 Noethterian for all i = 1, · · · , n − 1.
But we just proved that M i /M i+1 is Noetherian if and only if they are Artinian. Then by
exercise 16.1.6 the short exact sequence in Artinian version we can prove that R is Artinian.
The other direction is exactly the same argument.

64
Therefore we conclude that R is Noetherian if and only if R is Artinian.

65

You might also like